Kaplan Dermatology

¡Supera tus tareas y exámenes ahora con Quizwiz!

A 40-year-old formerly obese woman presents to her healthcare provider. She was very proud of having lost 80 lb during the previous 2 years, but has now noticed that her hair is "falling out." Upon questioning she reports having followed a strict, fat-free diet. Her alopecia is probably related to a deficiency of which of the following vitamins? A. A B. C C. D D. E E. K

The correct answer is A. . Although it is hard to develop a deficiency in oil-soluble vitamins (A, D, E, K) because the liver stores these substances, deficiency states can be seen in chronic malnutrition (specifically chronic fat deprivation) and chronic malabsorption. Vitamin A is necessary for formation of retinal pigments (deficiency can cause night blindness) and for appropriate differentiation of epithelial tissues (including hair follicles, mucous membranes, skin, bone, and adrenal cortex). Vitamin C (choice B), which is water soluble rather than oil soluble, is necessary for collagen synthesis. It presents most commonly with poor wound healing, bruising, and bleeding. Vitamin D (choice C) is important in calcium absorption and metabolism. It presents with bone pain, growth retardation, and muscle weakness and spasm. Vitamin E (choice D) is a lipid antioxidant that is important in the stabilization of cell membranes. Vitamin K (choice E) is necessary for normal blood coagulation. Clinical Pearls Vitamin A is a lipid-soluble vitamin, which is a retinoic acid Vitamin A is crucial to the cellular differentiation and integrity of the eye and therefore deficiency leads to dry eyes, fragility of the cornea, and clouding of the cornea. Vitamin A deficiency leads to night blindness. Dermatologically, vitamin A deficiency can lead to hyperkeratosis, follicular hyperkeratosis, and destruction of hair follicles with replacement with mucus-secreting glands. Chronic hypervitaminosis A can lead to alopecia, just as vitamin A deficiency can.

A 71-year-old man has had an indolent, nonhealing ulcer at the heel of the right foot for several weeks. The patient began wearing a new pair of shoes shortly before the ulcer started and noticed a blister as the first anomaly at the site where the ulcer eventually developed. He indicates that neither the blister nor the ulcer ever gave him any pain. The ulcer is 3.5 cm in diameter, the ulcer base looks dirty, and there is hardly any granulation tissue. The skin around the ulcer has a normal appearance. The patient has no sensation to pin prick anywhere in the affected foot. Peripheral pulses are weak but palpable. He is obese and has varicose veins, high cholesterol, and poorly controlled type 2 diabetes mellitus. Which of the following most accurately characterizes the ulcer? A. Diabetic ulcer due to trauma, neuropathy, and microvascular disease B. Ischemic ulcer due to arteriosclerosis C. Ischemic ulcer due to embolization D. Neoplastic in nature, probably squamous cell carcinoma E. Stasis ulcer due to venous insufficiency

The correct answer is A. Diabetic ulcers typically develop at pressure points, and the heel is a favorite location. The patient has evidence of neuropathy, and the correlation with the trauma inflicted by the new shoes is classic. Ischemic ulcers, whether due to arteriosclerosis (choice B) or embolization (choice C) are typically seen at the tip of the toes, as far away from the heart as one can get. Neoplasms (choice D) can indeed develop in long-standing ulcers, but the history would have been one of 10 or 20 years of healing and breaking down, before heaped up edges of cancer begin to develop. Stasis ulcers (choice E) are seen above the malleolus, surrounded by edematous, hyperpigmented skin. Clinical Pearls Diabetic ulcers are a multifactorial problem, which is usually caused by a combination of diabetic neuropathy, autonomic dysfunction, and vascular insufficiency. Non-ischemic neuropathic foot ulcers in a diabetic are usually the result of foot deformities and neuropathy, which prevents the sensation of pain in traumatized areas of the foot. Neuropathic diabetic ulcers are typically: Located in areas of repeated trauma (plantar metatarsal heads or dorsal interphalangeal joints. The result of hyperkeratotic overgrowth of tissue, such as corns or callouses on nontraumatic areas of the foot. The result of hyperkeratotic overgrowth of tissue, such as corns or callouses on nontraumatic areas of the foot. Seen in areas that lack sensation. Seen in patients with signs of neuropathy on physical examination.

A 32-year-old woman comes to the healthcare provider because of a persistent skin rash that appeared 2 months earlier. She first noticed a large, pink patch appear on her right flank, and then, about 10 days later, many smaller reddish, scaly spots started to emerge on her stomach, back, and shoulders. She tried using a topical steroid cream she found in her mother's medicine cabinet and that helped to stop the itch. However, the rash keeps coming back in the areas where her bra straps rub the skin and not only is it annoying but also it still itches. She has no other medical problems and takes no medication. In fact, she had a physical for a new job just before the rash started and was told that she is "in perfect health." Examination reveals an oval pink patch on her right flank and multiple pink macules on her trunk, especially on the back, where the longer axis of the lesions are aligned with the cutaneous cleavage lines. In the areas where her bra straps cause friction there are multiple juicy, salmon-pink papules with a scale at the periphery. Which of the following is the most likely diagnosis? A. Pityriasis rosea B. Psoriasis vulgaris C. Seborrheic dermatitis D. Tinea versicolor E. Varicella

The correct answer is A. Pityriasis rosea is a mild inflammatory exanthem of unknown etiology. It frequently begins with a single herald patch that is usually larger than succeeding lesions and may persist alone for a week or two before the other lesions appear. Individual patches are oval or circinate and covered with finely wrinkled, dry skin, or, when it desquamates, with a scale. By that time, involution of the herald patch has begun and the salmon-colored macular and papular discrete and coalescing lesions start fading away after a total of 3 to 8 weeks. The fully developed eruption has a "Christmas tree" pattern, with the longer axis of the oval macules and papules along the lines of skin cleavage. Treatment is symptomatic, but it is questionable whether treatment can shorten the natural duration of the illness. Ultraviolet B in erythema-dose exposures can be used to hasten involution after the acute inflammatory stage has passed. Topical steroids and oral antihistamines provide symptomatic relief to patients who experience pruritus, but do not affect the duration. Psoriasis vulgaris (choice B) is a chronic papulosquamous inflammatory skin disease that clinically present with well-demarcated, erythematous plaques covered with silvery-white, nonadherent scale. Typically, the elbows, knees, umbilicus, lower back, and scalp are affected, but the lesions may be diffusely distributed over the entire body or affect the entire skin in the form of psoriatic erythroderma. Psoriasis shows a familial tendency and is probably inherited in a polygenic manner, with variable expression and penetration. It may begin at any time but there are 2 peaks: in children age 10-12 and adults age 40-50. The course is chronic, with remissions and exacerbations. Seborrheic dermatitis (choice C) is an inflammatory skin disease that most commonly appears in teenagers or young adults as pruritic erythematous patches with greasy yellow or brown scale in the scalp (especially the frontotemporal hairline), supraorbital area, glabella, nasolabial folds, external ear canals, and anterior chest. In men, it often affects the sideburns and beard area. In women, it tends to appear in the inframammary region in addition to the above-named sites of predilection. The course is chronic, with remissions and exacerbations frequently triggered by emotional stress or intensive physical activity. As to the etiology, it has been postulated that the yeast Pityrosporon ovale contributes to the appearance of skin lesions, but the fact that a considerable number of patients do not improve on topical or oral antifungal medication indicates that other mechanisms of disease are implicated as well. Tinea versicolor (choice D) is a common skin infection with the yeast Pityrosporon ovale (Malassezia furfur) in young adults who perspire freely. Typical lesions consist of oval scaly macules, papules, and patches on the chest, shoulders, and back. On dark skin they often appear as hypopigmented areas, whereas on light skin they are slightly hyperpigmented. The expression of infection is promoted by heat and humidity. Treatment should be attempted with a topical antifungal, but if that fails, a single oral dose of ketoconazole or fluconazole should clear the infection. Pigmentary alteration will persist for a while until the infection finally resolves. Varicella (choice E), or chickenpox, is an infectious disease most commonly seen in children from countries where immunization for this disease is not routinely performed. It is caused by the varicella-zoster virus. It is characterized by maculopapules, vesicles, and crusts in various stages of evolution. Treatment is supportive and oriented toward prevention of superinfection of the skin lesions, with the goal of avoiding extensive scar formation. Clinical Pearls: Pityriasis rosea is a skin disease that is thought to have a viral etiology. This disease is benign and self-limited and begins with an eruption of a "herald" patch, which is a round or oval, sharply demarcated pink or salmon-colored lesion on the trunk. This herald patch gives way to smaller similar lesions on the trunk and upper extremities which have predilection to the lines of cleavage of the skin, leading to a Christmas tree pattern. Patients may have a prodromal illness, of headaches, fatigue, and pharyngitis, but this is rare and the most patients will only complain of pruritus. Treatment for the underlying cause is not necessary since the underlying cause is often elusive. Symptomatic treatment with antihistamines for itching may be provided. Another option is topical corticosteroids. Patients with extensive disease that leads to cosmetic concerns can be treated with phototherapy.

A 6-week-old girl comes to medical attention because of a bright red plaque on her neck. The plaque is slightly raised, measures 2 cm in greatest diameter, and blanches partially on pressure. The mother reports that the lesion appeared in the second or third week and has been gradually increasing in size. Which of the following is the most appropriate next step in management? A. No further diagnostic procedure or treatment is necessary B. MRI and/or CT studies C. Referral to a dermatologist for biopsy D. Treatment with corticosteroids or interferon-alfa E. Surgical excision

The correct answer is A. The gross appearance of this red plaque, combined with its development and further growth beginning in the first month of life, makes a diagnosis of hemangioma most probable. Hemangiomas may involve the skin or deep organs, and may be small or rather extensive lesions. Skin hemangiomas become apparent in the first month of life, being occasionally preceded by an erythematous macule. They grow for 6-10 months, reach a stable size, and then undergo spontaneous regression. The great majority of skin hemangiomas disappear entirely by age 9. Some hemangiomas need further diagnostic evaluation or treatment either because of their location, or because of the likelihood of being associated with other congenital anomalies. MRI and/or CT studies (choice B), for example, would be necessary in cases of extensive cervical hemangiomas, periorbital hemangiomas, or hemangiomas located in the lumbosacral region. These hemangiomas may be associated with underlying abnormalities and can extend into orbital or laryngeal structures. Referral to a dermatologist for biopsy (choice C) is rarely necessary when the gross features are characteristic of classic hemangiomas. Treatment with corticosteroids or interferon-alfa (choice D) may be indicated in cases of periorbital hemangiomas. These lesions may extend into the orbit and lead to progressive visual impairment. Ophthalmologic evaluation and follow-up is warranted. Surgical excision (choice E) is not recommended. Further growth of hemangiomas can be prevented or slowed by corticosteroids or interferon-alfa. Laser treatment is useful for ulcerated hemangiomas. Clinical Pearls Infantile hemangiomas are a common vascular tumor seen in infants and children. Caucasian female infants are more commonly affected. There is an increased risk in preterm infants. Lesions are not typically evident at birth, but will become apparent in the first couple of days to months after birth. Lesions can occur anywhere on the body, including mucous membranes and internal organs, but they do have a predilection for the skin of the head and neck. Size and depth of the lesions can be variable. When the lesions initially present, they will typically undergo a rapid proliferative phase that can last several months. This proliferative phase is followed by a spontaneous involution phase that usually begins after about a year and can last several years. Most hemangiomas are benign and require no treatment. Unfortunately, some hemangiomas can ulcerate and cause pain, bleeding, scarring, and/or infection. Larger hemangiomas can be a cosmetic concern.

A 55-year-old woman with long history of sunbathing develops an erythematous, slightly raised, 2-cm patch of skin on her cheek. The involved area has a rough, very adherent, yellow-brown scale. The lesion is treated with 2x daily application of 5-fluorouracil cream, for a total of 4 week-long cycles alternating with no treatment weeks. During this treatment, the lesion and surrounding skin initially become markedly erythematous but then eventually heal without scarring. Which of the following is the most likely diagnosis? A. Actinic keratosis B. Basal cell carcinoma C. Bowenoid papulosis D. Keratoacanthoma E. Malignant melanoma

The correct answer is A. The lesion described could be either an actinic (solar) keratosis or squamous cell carcinoma in situ (a condition known as Bowen's disease). While experienced dermatologists can reliably recognize many lesions as actinic keratoses, as opposed to squamous cell carcinomas or other malignant skin lesions, relatively inexperienced internists or other clinicians should probably have the lesions biopsied (a shave biopsy may be all that is needed) before treatment. Actinic keratoses occur on sun exposed skin areas, with favorite target sites including face, back of hands and arms, neck, bald heads, and back and chests of men who work without shirts. Treatment options include 5-fluorouracil (particularly good for large lesions or multiple lesions; does not cause scarring), cryotherapy (good for a few lesions; does not cause scarring), photodynamic therapy, and surgery (scarring, but obtains tissue for microscopic examination). Basal cell carcinoma (choice B) tends to have raised edges, and only partially responds to topical 5-fluorouracil, leaving cancer spreading below the skin surface because the cancer cells project deep into the dermis. Bowenoid papulosis (choice C) can cause multiple carcinomas in situ involving genital areas. Keratoacanthoma (choice D) cause a volcano-shaped lesion with a central keratotic center. Malignant melanoma (choice E) will show pigmentary color variations and irregular margins, and will not necessarily respond to 5-fluorouracil, as it can extend deep into the dermis. Clinical Pearls Actinic keratosis is a premalignant lesion for squamous cell carcinoma. It typically presents as an erythematous papule with an adherent scale and a rough texture in chronically sun-exposed skin. Treatment is topical therapy (5-fluorouracil, imiquimod), cryotherapy, curettage, photodynamic therapy, or excision.

A 30-year-old woman has chronic, silver-white, scaly patches with an erythematous border on the skin of her knees and elbows. Which of the following is the most likely etiology of this condition? A. Autoimmune disease B. Bacterial disease C. Fungal disease D. Granulomatous disease E. Large vessel vasculitis

The correct answer is A. The patient has psoriasis, a chronic disease that affects 1-2% of people in the United States. Psoriasis typically involves the elbows, knees, scalp, intergluteal cleft, penis, and lumbosacral area. Well-established lesions show extensive parakeratotic scales overlying a thinned or absent stratum granulosum. The mitotic rate in the underlying epidermis is increased, and mitotic figures can be seen well above the basal cell layer. Collections of neutrophils (Munro abscesses) can be seen within the parakeratotic stratum corneum. The pathogenesis of psoriasis is not well understood, but evidence has been accumulating that autoimmunity may play a role. It can occur in patients with other autoimmune diseases. Bacterial (choice B) or fungal (choice C) diseases may trigger psoriasis but are not the primary etiologies. Psoriasis is not associated with granuloma formation (choice D), but rather with the formation of microabscesses in the stratum corneum. Large vessel vasculitis (choice E) does not seem to play a role in the pathogenesis of psoriasis. Clinical Pearls Psoriasis is a chronic immune-mediated skin disease that causes erythematous papules or plaques with a silver scale. Peak incidence is ages 20-30 and also ages 50-60. It affects both sexes equally. Mild to moderate disease should be treated with topical steroids and emollients. More severe disease should be treated with phototherapy. Patients with a history of skin cancer or psoriatic arthritis should be treated with a systemic biologic agent that will control the active lesions and prevent progression of the disease. The classic x-ray finding of psoriatic arthritis of the distal digits is a pencil in cup deformity caused by the thinning of the distal ends of the phalanx. Psoriasis can also affect the nails and nail involvement is heralded by pitting of the nails.

During the month of December, a middle-aged patient with chronically dry skin develops widespread coin-shaped lesions. The lesions begin as itchy patches of vesicles and papules. These lesions later ooze serum and crust over. The lesions are most numerous on the extensor surfaces of the extremities and on the buttocks. The patient says that some of the lesions have appeared to heal and then reappear at the same sites. Which of the following is the most likely diagnosis? A. Nummular dermatitis B. Pompholyx C. Psoriasis D. Seborrheic dermatitis E. Stasis dermatitis

The correct answer is A. The patient most likely has nummular dermatitis, which is a chronic inflammation of the skin, the etiology of which is still unknown. The presentation illustrated in the question stem is typical. This condition classically is termed "coin-shaped" or "discoid" as a description of these skin lesions. Microscopically, the dominant feature is a localized spongiosis (corresponding to edema) of the epidermis, which may also contain minute fluid-filled holes that correspond to the tiny vesicles seen clinically in early lesions. Treatment of these patients is problematic, and numerous regimens involving corticosteroids or antibiotics have been recommended, each of which appears to work with some but not all patients. Pompholyx (choice B) produces deep-seated pruritic vesicles on the palms, fingers, and soles. Psoriasis (choice C) can produce coin-shaped lesions, but they are covered with silvery scale. Seborrheic dermatitis (choice D) produces hyperkeratosis on the scalp and face. Stasis dermatitis (choice E) can produce discoloration and ulceration of the lower legs near the ankles. Clinical Pearls Nummular dermatitis is a common dermatologic problem with an unknown cause that presents with intensely pruritic patches of eczematous dermatitis with areas of papules, scaling, crusting, and some serous oozing. Lesions vary in number and size: from a single lesion to multiple lesions that are usually circular and with measurements of 2-10 cm in diameter. Lesions are usually present on the trunk and lower extremities. The head is usually spared. Lesions usually appear spontaneously, and the underlying causes are not identified. Patients may give a history of exposure to a drying or irritating substance, such as excessive water, chlorine, or soaps prior to the development of these lesions. Differential diagnosis includes tinea corporis and xerotic dermatitis. The mainstay of treatment is a potent topical corticosteroid in emollient base. Systemic steroids in short courses are sometimes required. Antibiotics may be necessary if patients have superinfection. Patients should be encouraged to avoid irritants if they are identified as a cause. Patients should be told to moisturize the skin immediately after bathing.

A 46-year-old man comes to the healthcare provider because of an itchy rash that appeared in his groin over the previous several weeks and seems to be spreading. He has tried using over-the-counter "jock-itch" cream but it has not been effective. His medical history is significant for diabetes controlled with diet and hypertension for which he takes losartan (Cozaar). He has no known allergies to medications and the family history is negative for skin diseases and skin cancer. He does not drink or smoke. On examination there are sharply demarcated, dry, brown, slightly scaly patches in both genitocrural creases. There are similar but smaller patches in the axillae and in the fourth interdigital space on the feet. A Woods light examination is positive in all 3 locations. Which of the following is the most likely diagnosis? A. Erythrasma B. Inverse psoriasis C. Intertrigo D. Seborrheic dermatitis E. Tinea cruris

The correct answer is A. The typical clinical presentation of erythrasma is characterized by sharply delineated, dry, brown, slightly scaling patches occurring in the intertriginous areas, especially the axillae, the genitocrural creases, and the webs between toes 4 and 5. It is typically asymptomatic except for the groin lesions, which may burn or itch. The causative microorganism is Corynebacterium minutissimum. Woods light examination is the diagnostic test of choice because it shows a pathognomic coral-red fluorescence, which results from the presence of a porphyrin. Treatment is erythromycin. Topical erythromycin or clindamycin solution is easily applied and also very effective for more localized erythrasma. Inverse psoriasis (choice B) appears in intertriginous areas such as the sides of the neck, axilla, submammary folds, and intergluteal and genitocrural area, in the form of bright red, shiny, moist and glistening plaques with no scale. Sometimes there are tiny pustules at the edges of these plaques and there is usually chronic-type psoriasis in plaques on other body sites. It has a chronic course marked by remissions and exacerbations. Woods light examination will not show fluorescence in inverse psoriasis. Intertrigo (choice C) is an irritant contact dermatitis that appears in intertriginous areas due to heat, moisture, and friction. Shiny and moist pink patches are seen that at the base of the crease often have deep, hemorrhagic, and very painful fissures. Subjectively, stinging, burning, and pain are common symptoms. Seborrheic dermatitis (choice D) may appear in intertriginous areas but would be extremely rare in the interdigital space of the foot, because sebaceous glands are not at all prominent in that skin site. Dark red, moist plaques covered with yellowish, greasy scale, with or without fissuring, can be seen in the groin and intergluteal crease, as well as on the umbilicus and submammary region, and even in the axilla. Tinea cruris (choice E) is a common infection that may be caused by any of the dermatophytes that infect skin (Epidermophyton, Microsporum, and Trichophyton). Itchy, pink-to-red, scaly, annular, and polycyclic patches appear in the groin and spread centrifugally. In the toe webs (tinea pedis), dermatophytosis usually presents with thick, white, macerated, keratotic skin and a deep-seated fissure at the bottom of the crease. Woods light examination of tinea cruris and tinea pedis does not show fluorescence. Both infections are often seen in athletic persons who sweat extensively and use common locker rooms or swimming pools, through which the infection is readily spread. Clinical Pearls Erythrasma is a superficial skin infection caused by Corynebacterium minutissimum. Erythrasma can occur in anyone, but is more common in the presence of diabetes mellitus and other immune disorders. In fact, erythrasma may be the presenting symptom of diabetes or an immune compromising disease. Patients typically present with scaly, macerated skin in the interdigital spaces of the feet. Patients may have concomitant infection with candida or dermatophytes. The intertriginous form occurs as reddened or brown patches or plaques in the axilla, inframammary folds, umbilicus, or groin area. Lesions can be pruritic or asymptomatic. Wood's lamp examination reveals a classic coral red fluorescence.

A 40-year-old woman consults a healthcare provider about lesions on her neck that she finds cosmetically unattractive. On examination of the neck, multiple lesions that seem to be hanging off the skin are seen. Each lesion is small, soft, and pedunculated. The largest lesion is about 4 mm in diameter. The color of different lesions varies from flesh colored to slightly hyperpigmented. Which of the following is the most likely diagnosis? A. Acrochordons B. Lentigos C. Lipomas D. Seborrheic keratoses E. Spider angiomas

The correct answer is A. These lesions are skin tags, more formally known as acrochordons. They are very common benign lesions that can occur at any skin site, but have a predilection for the neck, axilla, and groin. Multiple lesions are common, and the lesions tend to increase in number with age. They are usually asymptomatic but can be irritating. Microscopically, an acrochordon consists of a fibrovascular core, sometimes also with fat cells, covered by an unremarkable epidermis. Asymptomatic skin tags are usually not treated. Many methods can be used to remove disfiguring or irritated skin tags, including freezing with liquid nitrogen, light electrodesiccation, or excision with scalpel or scissors. Lentigos (choice B) are flat macules characterized microscopically by an increased number of melanocytes occurring singly rather than in clusters in the lower part of the epidermis. Lipomas (choice C) produce soft, movable, subcutaneous nodules composed of mature adipose tissue. Seborrheic keratoses (choice D) are pigmented superficial epithelial lesions that are usually warty in consistency. Spider angiomas (choice E) are bright red, faintly pulsatile, vascular lesions of the skin. Clinical Pearls Patients should be warned about scarring, seroma, and possible hematoma formation after excision. Risk increases with age and about 25% of adults have skin tags. There is a familial predisposition for development of skin tags. Skin tags tend to appear in sites of friction, such as the axilla, neck, inframammary fold, and inguinal area. Skin tags become symptomatic when they are caught on clothing or jewelry. Diagnosis is clinical, and lesions need to be differentiated from neurofibromas and pedunculated dermal nevi. Skin tags are more commonly seen in obese individuals and these may be a sign of insulin resistance and metabolic syndrome. Treatment is indicated if the lesions become irritated or if they are cosmetically concerning for patients. Small skin tags can be removed without local anesthesia, but larger lesions should be anesthetized with lidocaine 1% with epinephrine or a topical anesthetic. Skin tags can be treated by excision with curved iris scissors, cryosurgery with liquid nitrogen, or with electrodessication. Larger skin tags may require suturing to control bleeding during excision. Silver nitrate, aluminum chloride, or electric cautery can also be used to control bleeding. Recurrence is not common, but new lesions can form in areas of friction.

A 14-year-old boy is brought to the office by his parents because of a skin infection that started on his face and is now spreading to other parts of the body. The first lesions occurred when he started school, a couple of months ago. An over-the-counter antibiotic cream has not helped, as new spots keep popping up. The child is otherwise healthy and has no medical problems. He does not take any medication on a regular basis and is up-to-date with his immunizations. His facial lesions are shown. In between these lesions are moist erosions covered with thick, friable, honey-colored crust. On the trunk, especially on the lateral sides and in the axilla, as well as on the extensor extremities, multiple areas with similar skin lesions are seen. Some of them have gyrate and annular patterns due to central clearing. Which of the following is the most likely diagnosis? A. Impetigo contagiosa B. Poison ivy C. Scabies D. Tinea corporis E. Toxic epidermal necrolysis

The correct answer is A. This boy has the typical clinical picture of impetigo contagiosa, a staphylococcal, streptococcal, or combined infection characterized by discrete, thin-walled vesicles that rapidly become pustular and then rupture. Impetigo occurs more frequently on the exposed parts of the body-the face, hands, neck, and extremities-although it may appear at sites of friction as well. Over 50% of cases are due to Staphylococcus aureus, with the remainder due to Streptococcus pyogenes or a combination of both bacteria. Group B streptococci are associated with newborn impetigo. Impetigo is most commonly seen in early childhood and during hot, humid summers in temperate climates Treatment includes skin hygiene with crusts being soaked off frequently along with application of a topical antibiotic such as 2% mupirocin. . Poison ivy (choice B) presents with edema and erythema of the eyelids and linear, erythematous, or brownish streaks that are studded with tense vesicles. Erythematous patches with vesicles and scale-crust may be present. Pruritus is usually excruciating. Scabies (choice C) presents with typical linear, 2- to 5-mm-long burrows, covered by fine scale and with a tiny vesicle at the end. They can most commonly be found in the finger webs, on the flexor wrists, areolas, axillae, or glutei, or around the umbilicus. The hypersensitivity rash that follows infestation is more widespread and consists of erythematous, excoriated, and crusted papules on the trunk and extremities. Incessant pruritus is typical and worsens at nighttime. Tinea corporis (choice D) typically present as annular erythematous patches that have a scaly and sometimes vesiculated edge. The center usually clears as the lesion spreads centrifugally. Multiple lesions can become confluent to form circinate and polycyclic patterns. In uncomplicated cases, there is no significant crusting apparent. A potassium hydroxide examination of skin scrapings is diagnostic. Toxic epidermal necrolysis (choice E) is a severe drug reaction where the entire or almost entire epidermis is sloughed off above the level of the lower spinous zone secondary to cytotoxic-induced necrosis of keratinocytes. Clinically, widespread erythema is present in a systemically severely ill patient and large sheets of skin slough off, leaving moist erosions. Mucosal involvement may lead to permanent sequelae in the form of conjunctival synechiae if strict ophthalmologic care is not maintained. Clinical Pearls Impetigo is a highly contagious superficial bacterial infection that usually involves the face and extremities. Lesions usually start as papules and then transform into vesicles or pustules and ultimately crust over. The most common cause of impetigo is Staph aureus, but it can also be caused by beta-hemolytic streptococci (primarily group A); some cases are caused by both staph and strep. Cases of impetigo can occasionally be followed by poststreptococcal glomerulonephritis or rheumatic fever. First line treatment for impetigo is topical mupirocin applied to the affected areas 3 times daily. Oral antibiotic treatment with dicloxacillin, cephalexin (Keflex), or clindamycin is recommended for bullous impetigo. If patients are high risk for MRSA or have a beta-lactam allergy, treatment with clindamycin or linezolid (Zyvox) is appropriate. All patients should be instructed on the importance of handwashing and skin hygiene to prevent the spread of impetigo.

A 37-year-old man consults a healthcare provider about multiple papules and plaques on his body. The lesions are widely disseminated across the body, with increased numbers on the trunk and arms. Mucosal surfaces are also involved. The lesions are barely elevated and vary in color from purple to pink to brown. Some of the lesions are irritated and have bled profusely. The lesions have been developing over the previous year. The patient thinks he may have also had blood in his urine and in his stool. This patient's condition is most likely to be related to which of the following conditions? A. AIDS B. Contact allergy C. Hemophilia A D. Melanoma E. Streptococcal infection

The correct answer is A. This is Kaposi sarcoma, which may be the presenting manifestation of AIDS. The lesions, which behave like a malignant vascular sarcoma, are caused by infection with a member of the herpes virus family (herpesvirus type 8). AIDS patients are generally very vulnerable to herpes virus infections, which also include herpes simplex I, herpes simplex II, herpes zoster-varicella, cytomegalovirus, and Epstein Barr virus. In Kaposi sarcoma, the virus infects endothelial cells in blood vessels, and causes formation of tumors that have spindle cells alternating with slit-like vascular spaces. The slit-like vascular spaces lack the normal mechanisms of vasoconstriction in response to injury, and consequently may bleed profusely after minor trauma. While the most obvious lesions are on the skin, similar lesions may be found in internal organs and may give rise to internal bleeding. Kaposi sarcoma has been treated with a variety of chemotherapies; isolated superficial lesions may alternatively respond to electrocoagulation, cryotherapy, and electron beam radiotherapy. Contact allergy (choice B) can cause erythema or blistering, but would not usually cause bleeding. Hemophilia A (choice C) can cause bleeding but would not cause persistent skin lesions other than (possibly) purpura. Melanoma (choice D) tends to produce lesions that may vary in color and may metastasize to the skin, but it would not be expected to be specifically associated with marked bleeding from the lesions unless late in the course of the disease. Streptococcal infection (choice E) can cause a disseminated skin rash, but this would not persist for a year and would not be a source of bleeding. Clinical Pearls Kaposi sarcoma is a cancerous lesion resulting from the abnormal angiogenesis of skin cells in patients with human herpes virus 8. Kaposi sarcoma is one of the AIDS-defining illnesses (along with pneumocystis jirovecii pneumonia and non-Hodgkin's lymphoma. Lesions are slow growing and are more common on the lower extremities, nose, oral mucosa, and genitals and are usually elliptical along the tension lines of the skin. The skin lesions are asymptomatic on the skin, but GI lesions may lead to weight loss, abdominal pain, nausea, vomiting, malabsorption, intestinal obstruction, diarrhea, and/or GI bleeding. AIDS-related Kaposi sarcoma is the most common neoplastic disorder in patients with HIV. These patients will typically present with a lesion on the hard palate and lesions will usually present when the CD4 counts are less than 200/mm3. Treatment of HIV with highly active antiretroviral therapy, HAART, has significantly reduced the incidence of AIDS-related Kaposi sarcoma.

A 72-year-old Vietnam veteran comes to the healthcare provider because of "barnacles" that have appeared on his scalp, face, and hands over the years. They have started to bother him lately because they keep catching on his clothes and he would like them to be treated in some way, if possible. He suffers from hypertension, diabetes mellitus, posttraumatic stress disorder, and a tobacco addiction. On examination there are multiple scaly, keratotic, erythematous macules and papules on his anterior scalp, forehead, temples, nose, forearms, and dorsal hands. Their size ranges from 2 to 10 mm in diameter. Which of the following was the most significant contributing risk factor in the development of these skin lesions? A. Agent Orange exposure B. Excessive sun exposure C. Diabetes mellitus D. Posttraumatic stress disorder E. Tobacco addiction

The correct answer is B. This patient has actinic keratoses, and the most significant contributing risk factor in their development was excessive sun exposure. Actinic keratoses may occur on any chronically or repeatedly sun-exposed part of the body. They are usually multiple, discrete, flat or elevated, verrucous or keratotic, red, pigmented, or skin-colored. The surface is commonly covered by an adherent scale. They may develop as early as the twenties and thirties in fair-skinned people who live in areas with high ultraviolet exposure and do not use sunscreens for protection. Clinical signs of photodamage, such as solar lentigines, poikiloderma of the neck, and facial telangiectasia, are strong risk factors for the development of solar keratoses and skin cancers. Histopathologically, they show atypical keratinocytes consistent with early squamous cell carcinoma in situ and 90% show p53 chromosomal mutations consistent with squamous cell carcinoma. Therefore, actinic keratoses require appropriate treatment and adequate patient education. Physical methods of destruction (liquid nitrogen) or topical chemotherapy (5-fluorouracil) are effective treatments. Patients should be advised to refrain from excessive sun exposure and to apply sunscreen liberally and reapply frequently when exposed to the sun. Agent Orange exposure (choice A) has not been associated with the appearance of actinic keratoses, nor any other specific skin condition. However, the veterans who fought in World War II and Vietnam did suffer from excessive sun exposure. Unfortunately, many of these now-elderly people have severe sun damage of the skin with greatly enhanced potential for development of skin cancers. Diabetes mellitus (choice C) is associated with the appearance of well circumscribed, hyperpigmented, atrophic macules and patches on the anterior shin that are called diabetic dermopathy consistent with acanthosis nigricans and are the result of microangiopathic changes in the skin and additional minor trauma. In addition, they may get yellow-orange, indurated, shiny plaques on the lower extremities that are called necrobiosis lipoidica and are significantly associated with diabetes but do not follow the course of the disease. Diabetic patients are not more prone to the development of actinic keratoses by nature of their disease. Posttraumatic stress disorder (choice D) does not have any specific skin changes but may be associated with secondary lesions, such as excoriations, postinflammatory dyspigmentation, or scarring as a result of neurotic excoriations. Tobacco addiction (choice E) may be associated with a less vibrant, dull grayish skin tone, nicotine stomatitis (red papules on the soft palate), leukokeratosis of the lip, and squamous cell carcinoma of the lip and oral mucosa, but it is not a risk factor for the development of actinic keratoses. Clinical Pearls Actinic keratoses (AKs) are common skin lesions that form from the hyperproliferation of atypical epidermal keratinocytes. The greatest risk factor for their development is excessive sun exposure, but having fair skin, an advanced age, and being male are also known risk factors. The lesions typically present as a scaly, erythematous macule or papule in areas of sun exposure (scalp, face, lateral neck, dorsum of hands and forearms). AKs can progress to squamous cell carcinoma (SCC). Treatment options include topical medications, photodynamic therapies, destructive therapies, and surgical excision.

A 50-year-old man develops a mass on the back of his hand. The lesion somewhat resembles a "volcano" and consists of a round, firm, flesh colored, 1-cm nodule with sharply rising edges and a central crater. Keratotic debris can be expressed from the central crater. The lesion has developed very rapidly over about a 3-month period. Which of the following is the most likely diagnosis? A. Keratoacanthoma B. Lipoma C. Malignant melanoma D. Pyogenic granuloma E. Seborrheic keratosis

The correct answer is A. This is a keratoacanthoma. These lesions have the distinctive appearance described in the question stem, and tend to occur on sun-exposed areas including the face, the forearm, and the dorsum of the hand. These lesions microscopically closely resemble squamous cell carcinoma, but are now considered benign. They may resolve spontaneously, and they may not need resection. However, they are usually resected since both patient and healthcare provider often feel uncomfortable with neglecting them without having a definitive diagnosis and being absolutely certain they will spontaneously regress. Lipoma (choice B) causes a soft subcutaneous mass. Malignant melanoma (choice C) usually causes a pigmented lesion, often with feathery edges suggesting invasion. Pyogenic granuloma (choice D) causes a red mass, often at a site of previous injury. Seborrheic keratosis (choice E) causes a warty lesion most often found on the temple or trunk. Clinical Pearls Keratoacanthomas are rapidly growing keratocytic epithelial tumors that are usually found on sun-exposed areas of the skin. Keratoacanthomas usually present as a dome-shaped or crater-like nodule with a central keratotic core that develops over a few weeks.

A 35-year-old woman comes to a clinician because of an itchy patch of skin near her knee. 6 months previously, she had had several mosquito bites in the relatively small area. The mosquito bites eventually resolved, but the area had continued to be itchy, and she had continued to scratch it periodically. The well-defined 6-cm diameter patch of skin is now dry, scaling, hyperpigmented, and thickened. A ring of discrete brownish papules can be seen at the periphery of the lesion. Which of the following is the most likely diagnosis? A. Lichen simplex chronicus B. Pompholyx C. Psoriasis D. Seborrheic dermatitis E. Stasis dermatitis

The correct answer is A. This is lichen simplex chronicus, also known as localized scratch dermatitis and neurodermatitis. What basically happens is that the patient gets into a vicious cycle of itching causing scratching, which causes more itching. The initiating cause may or may not be known or remembered, but the process persists long after the inciting process has resolved. Stress and tension may exacerbate the process. Treatment involves breaking up the itch-scratch cycle, which may be difficult. Topical steroids are effective, as is covering the area to make scratching more difficult. Pompholyx (choice B) means "bubble" and is a cause of marked pruritic vesicles of the fingers, palms and soles. Psoriasis (choice C) causes salmon-colored plaques with silvery scale. Seborrheic dermatitis (choice D) causes dandruff and cradle cap. Stasis dermatitis (choice E) is seen at the ankles and is related to varicose veins. Clinical Pearls Lichen simplex chronicus is a disease of tissue damage caused by deliberate and repetitive manipulation of the skin. Pathologic skin picking (PSP) is repetitive picking, scratching, rubbing, or squeezing of the skin, leading to visible skin damage and impaired social functioning. Damage may be caused by fingernails, teeth, scissors, tweezers, or pins. PSP ultimately leads to lichen simplex chronicus with excoriated, polymorphic lesions. There may be areas of hypo- or hyperpigmentation. Newer lesions may have a serosanguineous crust whereas older lesions may be crusted or appear as hypertrophic nodules or atrophic scars. Complications of PSP include ulcerations, infections, and scarring.

A 42-year-old man comes to the emergency department complaining of a painful infection on his right buttock that is interfering with his daily activities. In addition, it has been draining pus for the past 24 hours and seems to be getting worse, despite the fact that it has "opened up." He is in good health and takes an occasional sleeping pill. He does not smoke, but drinks a bottle of wine daily with dinner. As a mechanic, he is frequently exposed to dirt and machine oils and often gets boils on his neck and thighs, especially in the summer when it's hot and humid. On physical examination there is an erythematous and edematous 6 x 8-cm indurated plaque on the right buttock, which has within it multiple necrotic, suppurating follicular ostia. The plaque is ill-defined and blends into the surrounding skin with induration palpable beyond the erythematous edge. Which of the following is the most likely diagnosis? A. Carbuncle B. Epidermal cyst C. Folliculitis D. Furuncle E. Herpes zoster

The correct answer is A. This patient has a carbuncle, a collection of multiple confluent furuncles with separate heads. A furuncle (choice D), or a boil, is an acute, round, tender, circumscribed, perifollicular staphylococcal abscess that generally ends in central suppuration. The lesions begin in hair follicles and continue for a prolonged period through autoinoculation. Several adjoining furuncles will end in a painful, erythematous, and edematous plaque with multiple heads called a carbuncle. The lesions undergo central necrosis and rupture through the skin, in addition to forming channels below the skin by which the abscesses communicate. Purulent, necrotic debris is discharged. Common sites are the nape, axillae, and buttocks. An epidermal cyst (choice B) is an often fluctuant, tense swelling varying from 0.5 to several centimeters in diameter. The surface of the lesion is smooth and shiny from the underlying pressure. These nodules are freely movable in relation to underlying structures and are attached to the overlying skin by remnants of the dilated gland duct. A central punctum is present on the surface in the form of a comedo and is the opening of the affected duct. Macerated keratin and cheesy sebaceous material comprise the cyst contents. A foul smell emanates from that material if it is expressed through the opening or if the cyst wall is severed during surgical extirpation. It will recur if it is incompletely excised. Folliculitis (choice C) is a superficial staphylococcal infection of hair follicles that presents as small, thin-walled pustules at the follicular openings. Most commonly, they appear on the scalp or extremities, as well as periorally. These fragile, yellowish-white, domed pustules develop in crops and heal in a few days. Staphylococcus aureus is the most common cause, but other bacteria may cause similar clinical findings in the appropriate setting. Herpes zoster (choice E), or shingles, is caused by varicella-zoster virus. Clinically, pain may precede skin lesions by several days. Once the skin changes occur they are usually located along a dermatome, most commonly on the chest or the first trigeminal branch area of the face and scalp, in the form of grouped vesicles on an erythematous base that extends from the posterior midline anteriorly, outlining that dermatome. Clinical Pearls An abscess is a collection of pus within the dermis or deeper skin structures. A "boil" or furuncle is an infection of the hair follicle in which the purulent material extends into the subcutaneous tissues and forms an abscess. A carbuncle occurs when multiple furuncles combine to form several inflamed follicles that develop into a single inflammatory mass with purulent drainage. Risk factors for the development of these lesions include diabetes mellitus, immune-suppressing diseases, and any breach in the skin integrity, though healthy individuals can also develop them. The lesions can be polymicrobial or monomicrobial. Staph aureus is present in up to 50% of cases. Treatment includes warm compresses to promote drainage. If there is fluctuance of the lesions, an incision and drainage (I and D) should be performed with attempts to break up all loculations within the abscess. The role of antibiotics in addition to I and D is controversial, but may be beneficial. Patients with multiple lesions, surrounding cellulitis, immune compromised states, and systemic signs of infection and should be treated with both I and D and systemic antibiotics. In other patients, I and D may be sufficient.

A 42-year-old African American man is hospitalized for edema of the left lower leg associated with fever, chills, and malaise. The swelling started a day earlier and suddenly became intensely red, hot, and painful to the point that he could not walk. He does remember injuring his shin a couple of days before the swelling began. The patient's medical history is significant for type 1 diabetes treated with subcutaneous insulin, hypothyroidism treated with levothyroxine supplementation, and mild intellectual disability. He is very conscientious about his medication and lives with his parents, who make sure he does not miss any doses. On physical examination the patient is in mild acute distress. His temperature is 38.7° C (101.6° F), blood pressure 125/75 mm Hg, pulse 100/min, and respirations 22/min. Fasting blood glucose is within normal limits and hemoglobin A1C 6%. The left lower leg has diffuse, ill-demarcated, pitting edema, erythema, and an increase in local temperature over the distal half. There is a central painful, nodular component with ulceration and discharge of purulent material. The left popliteal and inguinal lymph nodes are palpable. Which of the following is the most likely diagnosis? A. Cellulitis B. Diabetic dermopathy C. Erysipelas D. Necrobiosis lipoidica E. Necrotizing fasciitis

The correct answer is A. This patient has developed cellulitis following an injury of the shin. Cellulitis is a suppurative infection of the dermis and subcutaneous tissue most commonly caused by Staphylococcus aureus or Streptococcus pyogenes. Usually, it follows some discernible wound. Complications include gangrene, metastatic abscesses, and sepsis. Initial empiric therapy should cover both streptococci and staphylococci in expectation of culture and sensitivity studies. IV penicillinase-resistant penicillin or a first-generation cephalosporin is commonly effective. Any necrotic material should be surgically debrided and local wound care performed. Early treatment is important to prevent significant lymphatic compromise and subsequent increased risk of recurrence and elephantiasis. If any residual edema persists it is crucial to prevent even minor trauma to the area, as it will almost certainly result in recurrent episodes of cellulitis. Diabetic dermopathy (choice B) presents as well circumscribed, hyperpigmented, atrophic macules and patches on the anterior shins of patients with long-standing poorly controlled diabetes. They are the result of microangiopathic changes in the skin and additional minor trauma. Erysipelas (choice C) is a skin infection that has a distinct border and is typically caused by group A strep. Treatment is oral penicillin. Necrobiosis lipoidica (choice D) is a granulomatous skin disease of unknown etiology that is most commonly seen in patients with long-standing diabetes but does not follow the course of their disease. Yellow-orange, indurated, shiny plaques appear on the lower extremities and, with time, resolve leaving atrophic patches with thin skin and no adnexal structures. They are asymptomatic but conspicuous skin lesions for which there still is no treatment consistently proven to be effective. Necrotizing fasciitis (choice E) is an acute necrotizing infection involving the fascia. It may follow surgery, perforating trauma, or appear de novo. Within 24 to 48 hours pain, erythema and edema progress to patches of dusky blue discoloration with or without serohemorrhagic blisters. Anesthesia of the involved skin is very characteristic. By day 4 or 5, the purple areas have become gangrenous. Magnetic resonance imaging is indicated to delineate the extent of deep involvement. Early surgical debridement and IV antibiotics are the only chance of surviving this virulent and aggressive infection, which carries 20% mortality in the best of circumstances. Clinical Pearls Cellulitis and erysipelas are infections of the skin, characterized by erythema, swelling, and warmth with no underlying suppurative foci. Erysipelas tends to be raised with a clear demarcation from the surrounding skin. Both infections are usually caused by some disruption to the skin barrier, such as trauma, inflammation, pre-existing skin infection, or edema. Both are predominantly caused by beta-hemolytic streptococcus. Cellulitis can also be caused by Staph aureus or gram-negative aerobic bacilli. Cellulitis with no purulent drainage /exudates and no associated abscess, known as nonpurulent cellulitis should be treated with empiric antibiotics to cover beta-hemolytic streptococci and methicillin-susceptible Staph aureus (MSSA). If a patient has known history of MRSA or risk factors for MRSA, treatment with antibiotic that covers both MRSA and streptococci should be initiated (clindamycin, amoxicillin + Bactrim, amoxicillin + doxycycline, or linezolid). Cellulitis with associated purulent drainage/exudates with no drainable abscess ("purulent cellulitis") should be treated empirically for presumed MRSA until the cultures are complete.

A 6-year-old boy is brought to the emergency department because of a painful and swollen right forearm. He was bitten and scratched by a family cat 2 days ago in the affected area. His temperature is 39.6 °C (103.2 °F). The right forearm is erythematous, edematous, and tender to touch. He is not allergic to any medications. Which of the following is the most appropriate antibiotic treatment for this patient? A. Ampicillin B. Amoxicillin-clavulanate (Augmentin) C. Clindamycin D. Tetracycline E. Trimethoprim-sulfamethoxazole (Bactrim)

The correct answer is B. Animal bite is a common problem in pediatrics. In this case, the patient was bitten by a cat. To initiate appropriate antibiotic treatment, one needs to understand which organisms are most likely causing this infection. In a cat-bite wound, the most common organisms isolated are Pasteurella multocida and Staphylococcus aureus. Among the choices, amoxicillin-clavulanate (Augmentin) is the only antibiotic that is effective against both organisms. P. multocida infection usually manifests within 24-48 hours following the bite or scratch as localized swelling, erythema, tenderness, and serous or sanguinopurulent discharge. Fever, chills, and lymphadenopathy can also occur. Complications include tenosynovitis, osteomyelitis, and septic arthritis. P. multocida is found in the oral flora of 70% to 90% of cats, 25% to 50% of dogs, and a variable percentage in other animals. S. aureus is a common pathogen for cellulitis. It is often found on the skin. In a severe case of animal bite, the pathogens are usually polymicrobial. Therefore, amoxicillin-clavulanate is the treatment of choice; 7-10 days therapy is usually sufficient. Ampicillin (choice A) is an effective treatment against P. multocida, but it has no effect on S. aureus. Clindamycin (choice C) is effective against most serotypes of S. aureus, but is ineffective against P. multocida. Tetracycline (choice D) is effective again P. multocida, but it should not be given to children younger than 8 years. Trimethoprim-sulfamethoxazole (choice E) is not adequate on its own. When combined with clindamycin, however, it is an alternative treatment if the patient is allergic to penicillin compounds. Clinical Pearls Animal bite wounds tend to become infected with the oral flora of the biting animal (Pasteurella, Capnocytophaga, and anaerobes) as well as the skin flora of the victim (staphylococci and streptococci). Cat bites are predominantly infected by Pasteurella multocida and patients will typically have fever, swelling, erythema, tenderness, purulent drainage and lymphangitis. More serious complications of cat bites include abscesses, osteomyelitis, septic arthritis, tendonitis, and bacteremia. Treatment of all bite wounds should include copious irrigation with sterile saline and removal of all visible debris. Hand bites may warrant a surgical evaluation. Infected bite wounds should not be repaired and should remain open after debridement. The only exceptions to this are facial wounds, large bleeding lacerations, and disfiguring lacerations, which all should be closed primarily. Treatment should also include empiric antibiotics to cover the most common organisms likely to cause infection. Large or deep wounds should initially receive IV antibiotics and then converted to oral antibiotics. Augmentin (amoxicillin-clavulanate) is the most commonly used antibiotic in non-PCN allergic patients and has been shown to decrease the incidence of infections after animal bites. All patients should be questioned about vaccination status against tetanus and rabies.

A pediatric healthcare provider's office gets a phone call from a frantic mother. Her 7-year-old daughter was playing under the sink and accidentally spilled a strong, corrosive alkaline drain cleaner all over her arms and legs. The nurse on the phone can hear the girl screaming in the background. The healthcare provider's office and nearest hospital are a few miles away. Which of the following are the most appropriate instructions to give the mother? A. Cover the burned areas with triple antibiotic ointment until the girl can be seen at the office B. Get the girl into the shower right away and keep the water running over her for 30 minutes before bringing her to the emergency department C. Get the girl to the emergency department as soon as possible D. Wash the burned areas with diluted vinegar and bring the girl to the office E. Wrap the burned areas in sterile dressings before bringing the girl to the emergency department

The correct answer is B. By far the most important thing to do for a caustic chemical burn is to wash away the caustic agent as soon as possible. The best way to do this is via massive irrigation. Any answer that allows the chemical agent to stay in touch with the skin, whether mixed with antibiotic ointment (choice A), wrapped in bandages (choice E), or with no specific additional instructions (choice C) will result in continued burning for the time that it will take to get to the emergency department or healthcare provider's office. Washing with an acid (choice D) is particularly inadvisable. With the exception of chemical burns to the esophagus, massive irrigation should be started as soon as possible at the site of injury with tap water. One should, however, never "play chemist" to neutralize the agent when dealing with alkaline or acid burns. The chemical reaction will generate heat and compound the problem. Clinical Pearls Damage to tissue with chemical burns is a product of the potency, concentration, and length of contact with the toxic agent. In general, all chemical burns should be treated with: Prevent exposure of rescuers and healthcare workers to the chemical Patient should be removed from the area where the exposure occurred Remove all clothing and accessories from the patient Dry chemicals should be brushed off the patient (do NOT use bare hands) Copious irrigation with water to dilute and remove the majority of the chemical - "the solution to pollution is dilution" Tissue destruction will continue for as long as the chemical is in contact with the patient. Once the epidermis is damaged, toxins can more easily penetrate the permeable dermis and can lead to systemic toxicity. Knowledge of the chemical is crucial before copious irrigation is initiated as some chemicals can cause an exothermic reaction when mixed with water. This can lead to a thermal burn in addition to the chemical burn or could release toxic byproducts, which could be inhaled.

A 3-year-old girl is brought to the healthcare provider by her parents because of newly developed skin lesions that appeared on her neck and in the armpits over the last several weeks. She had just started going to day care when the teachers pointed out the "pimples." She is otherwise healthy and has no medical problems except for mild childhood eczema that is well controlled with daily emollients and over-the-counter cortisone preparations. The parents have noticed, though, that the girl has been scratching in the areas with skin lesions more often than usual and some of the lesions have become red and swollen. On physical examination, this is a pleasant and happy 3-year-old who is joyfully playing with a toy. She is well developed and well nourished. On her lateral neck and in both axillae, extending down the sides of her trunk, there is a total of approximately 20 flesh-colored, pearly papules with a central umbilication. Some of them have an erythematous halo and are crusted. Which of the following is the most likely diagnosis? A. Herpes simplex B. Molluscum contagiosum C. Varicella D. Verruca plana E. Verruca vulgaris

The correct answer is B. This patient has molluscum contagiosum, an infection caused by a poxvirus that typically affects children and teenagers but can also be seen in young adults and immunosuppressed patients. The mode of transmission is through direct contact. Frequently, small epidemics are seen in child-care facilities or schools, or among people who engage in contact sports or through sexual encounters. The typical skin lesion is a flesh-colored or grayish pearly papule 1 to 5 mm in diameter with a central umbilication through which grayish material is sometimes extruded. Treatment is by methods of physical destruction, such as cryotherapy with liquid nitrogen or curettage. Topical retinoids are also used (and must be applied precisely to the lesions with a cotton-tipped applicator) as they initiate an inflammatory reaction and resolution. Herpes simplex (choice A) clinically appears as a well-demarcated group of translucent, thin-roofed vesicles that tends to recur in the same location. It is caused by herpes simplex virus types I or II. The primary infection is very exuberant and painful, with widespread blistering of the affected mucosa, pain, and systemic symptoms. After the primary infection the virus becomes latent in cranial or spinal ganglions, only to reappear in the above-described form after certain triggering factors are met. Most commonly, those are illness, stress, or physical trauma. The recurrent episode tends to persist for 7 to 14 days and resolve without scarring. Varicella (choice C), or chickenpox, is an infection caused by the varicella-zoster virus. It most commonly affects children between the ages of 5 and 9; however, it can occur at any age. It is characterized by a rash (maculopapules, vesicles, and scabs in various stages of evolution), low-grade fever, and malaise. Verruca plana (choice D) is the plane wart, an infection caused by human papillomavirus type 3 that affects children and young adults. It is also commonly seen in immunocompromised patients. Multiple flesh-colored, tan, or pink flat papules are most often seen on the face and dorsa of the hands. They appear in crops and may last for many months or years before spontaneously resolving. Topical treatments with retinoids or physical methods of destruction such as topical salicylic acid are used for treatment. Verruca vulgaris (choice E), or common wart, is a ubiquitous infection seen in children and young adults that is caused by human papilloma virus, most frequently types 1, 2 and 4. Clinically, it presents as a gray or pink papule with a verrucous (cauliflower-like) surface. The most common location is the hands, although common warts may appear on any part of the skin. Liquid nitrogen application, curettage with electrodessication, surgical excision, and potent topical keratolytics (such as 20 to 40% salicylic acid applied strictly to the wart) are treatment options. Clinical Pearls Molluscum contagiosum is a usually self-limited infection caused by a poxvirus which presents as flesh-colored papules with a central umbilication. It has a variable incubation period of 2-6 weeks. It is usually seen in children, but can occur in any age group as is transmitted via direct skin contact or through fomites. Genital molluscum contagiosum can be transmitted sexually. Patients who are immune compromised may develop larger lesion or more diffuse disease. Diagnosis is usually clinical, but biopsy can be performed to confirm the diagnosis in which eosinophilic cytoplasmic inclusion bodies are found within keratinocytes. Most cases require no treatment. Genital molluscum should be treated to prevent further sexual transmission. Treatment with cryotherapy, curettage, cantharidin, or podophyllotoxin is recommended. Hypopigmentation may result from cryotherapy in dark skinned patients. Cantharidin should not be applied to the genital region. Podophyllotoxin safety is not established in children. Some healthcare providers have also treated molluscum with imiquimod, potassium hydroxide, salicylic acid, and topical retinoids, but all of these require weeks of treatment and patients would likely improve in that same time frame even without treatment. Patients with molluscum should not be excluded from work or school, but lesions should be covered with clothing or bandages to prevent transmission.

A 4-year-old girl presents with a maculopapular rash on her hands and feet and painful ulcers distributed anteriorly on her lips, palate, tongue, and buccal mucosa. Systemic features and lymphadenopathy are absent. Which of the following viruses is most likely to have caused this disorder? A. Coronavirus B. Coxsackievirus type A16 C. Herpes simplex virus type 1 D. Parainfluenza type 3 E. Rhinovirus

The correct answer is B. Hand-foot-and-mouth disease is characterized by the appearance of ulcers in the mouth and a maculopapular or vesicular rash on the hands and feet. It is most frequently caused by coxsackievirus type A16, although other coxsackieviruses have occasionally been implicated. The disease usually affects young children. Systemic features and lymphadenopathy are absent, and recovery is uneventful. Coronavirus (choice A) is a cause of the common cold. Herpes simplex virus type 1 (choice C) causes a variety of diseases, including gingivostomatitis, pharyngotonsillitis, herpes labialis, genital herpes, keratoconjunctivitis, and encephalitis. It may cause painful ulcers of the oral region, but it is unlikely to have caused the maculopapular rash described in this question. Parainfluenza virus (choice D) is responsible for croup, which is an acute febrile illness with stridor, hoarseness, and cough. Rhinovirus (choice E) referred to as the common cold, is a member of Picornaviridae. It is the most common cause of this cold. Clinical Pearls Hand-foot-mouth disease (HFMD) is an acute viral illness that is most commonly seen in children. It presents with fever, oral vesicles, and tender cutaneous lesions on the hands and feet, and less commonly, genitalia and buttocks. This viral infection is most commonly caused by the Coxsackie A virus with Coxsackie A16 causing the majority of cases of HFMD in the USA. The incubation period for the illness is 3-5 days and the acute illness will usually resolve in 2-3 days without complications. HFMD caused by enteroviruses can lead to CNS involvement. HFMD is contagious and spreads by direct contact with nasal discharge, saliva, blister fluid, or stool of infected patients, especially during the first week of the illness.

A 40-year-old woman is evaluated by a dermatologic healthcare provider because she has many pigmented lesions on her body. Examination of the skin and scalp demonstrates over 100 individual lesions, most of which vary in size 5-12 mm. They are found all over her body, but most commonly on sun-exposed skin. They are predominately round in shape, but some have subtly notched borders or are slightly asymmetrical. The woman's entire skin surface is photographed, and when the photographs are repeated 6 months later, no change in appearance of any of the lesions is noted. Which of the following is the most likely diagnosis? A. Compound nevi B. Dysplastic nevi C. Halo nevi D. Lentigos E. Malignant melanomas

The correct answer is B. Pigmented, mole-like lesions are very common in the general population, and virtually every healthcare provider needs to develop some skill in distinguishing obviously benign lesions from potentially malignant ones. Dysplastic nevi are an intermediate category between obviously benign nevi and malignant melanoma. While not considered cancerous themselves, they do have an increased rate of progression to melanoma. Isolated dysplastic nevi are often excised to remove the melanoma risk. More problematic are cases like this one, in which large numbers of dysplastic nevi are present. In this situation, careful monitoring with serial photographs can identify any changing lesions, which may be undergoing malignant transformation. Compound nevi (choice A) are usually dark, typically elevated, 3 to 6 mm lesions with a very regular shape; most people have about 10 of these lesions. Halo nevi (choice C) are flesh-colored or dark nodules, usually 3 to 5 mm, surrounded by a ring of depigmented skin. Lentigos (choice D) are flat, sharply marginated, uniformly pigmented, 2 to 4 mm diameter skin lesions. Malignant melanomas (choice E) are often quite asymmetrical in shape and the color varies more widely than in dysplastic nevi, potentially showing tan, brown, black, blue, red, or clear areas. Clinical Pearls Atypical nevi are melanocytic nevi that share some of the clinical features of melanoma (asymmetry, irregular borders, color variations, and diameter greater than 6 mm). Atypia is defined as: Color variation: pink, tan, brown, dark brown all within the same lesion Lesions with flat and elevated components or a "pebbly surface" Poorly defined borders that are irregular Larger size: 4-12 mm (size is the least reliable criteria for an atypical lesion) Although the lesions are benign, there is increased risk of development in to melanoma, especially when there are multiple lesions, family history of melanoma, personal history of melanoma, and excessive sun exposure.

A 6-week-old boy is brought to the healthcare provider because of a rash involving the diaper area. The mother has 5 other children at home and says that she is very busy 'tending to all of them'. On examination, there are erythematous, slightly scaly patches covering the buttocks and the lower abdomen. Skin creases appear spared. The baby is otherwise healthy. Physical examination reveals no lymphadenopathy, fever, or other signs of organic illness. Which of the following is the most common cause of this condition? A. Candidiasis B. Irritant contact dermatitis C. Langerhans cell histiocytosis D. Psoriasis E. Seborrheic dermatitis

The correct answer is B. The clinical presentation is consistent with diaper dermatitis. The most important clinical clue is the distribution of the erythematous areas. The usual cause of diaper dermatitis is irritant contact dermatitis, resulting from the concomitant action of fecal and urinary enzymes, entrapped moisture and excessive heat in the diaper area. Avoiding tight occlusion by diapers, bathing with mild soaps and water, and frequent diaper changes constitute the essential approach to treatment. In the most severe cases, topical application of low-strength corticosteroid cream and/or zinc oxide may be indicated. Candidiasis (choice A) manifests with papules and plaques rather than extensive shiny areas of erythema. Candida infection, however, develops superimposed on diaper dermatitis. Satellite lesions occurring outside of the primary rash helps to make this diagnosis. If signs of candidiasis are present, a topical antifungal agent should be used in treatment. Langerhans cell histiocytosis (choice C) is a severe disease due to neoplastic proliferation of Langerhans cells. When this condition is present, the baby is critically ill with fever and a diffuse scaly rash. Psoriasis (choice D) may be confused with diaper dermatitis. Psoriasis is characterized by well-demarcated silvery plaques that often involve skin folds in infants. Seborrheic dermatitis (choice E) manifests with salmon-colored, scaly, oily plaques that may affect the diaper area, but most commonly involve the scalp and face. Clinical Pearls Diaper dermatitis is a generic term that refers to numerous skin disorders in the diaper area. Irritant diaper dermatitis involves the convex surfaces of the diaper areas and spares the skin folds, unless there is also a candidal infection. Lesions are erythematous and may be asymptomatic or they can me painful papules with erosions. First-line therapy is prevention, with frequent diaper changes. Topical barrier ointments are preferred to powders as powders can lead to respiratory problems. Patients with concomitant candidal infection, or patients who have had symptoms for >3 days, should be treated with a topical antifungal. Low-potency nonhalogenated topical corticosteroids (1% hydrocortisone) can be added for severely inflamed irritant diaper dermatitis and should be applied sparingly twice daily for no more than 7 days. Topical corticosteroids can be used in conjunction with topical barrier ointments and/or antifungals. Combination topical corticosteroids/antifungal creams should be avoided, as the steroids are too potent for infant skin and can lead to skin atrophy and/or adrenal suppression.

A 71-year-old West Texas farmer of Irish ancestry has a nonhealing, indolent, punched out, clean-looking 2-cm ulcer over the left temple. The ulcer has been slowly growing over the past 3 years. There are no enlarged lymph nodes in the head and neck. He comes to the office now for evaluation. Which of the following would best dictate proper management? A. Full thickness biopsy of the center of the lesion B. Full thickness biopsy of the edge of the lesion C. Pathologic studies after the entire lesion is resected with a margin of 1 cm of normal skin all around D. Response to a trial of radiation therapy E. Scrapings and culture of the ulcer base

The correct answer is B. The history (a fair skinned person who is out in the sun all day) suggests a skin cancer, and the location (the upper part of the face) favors a basal cell cancer but does not exclude a squamous cell carcinoma, or even a melanoma. Thus, diagnosis is needed before proper therapy is instituted. The edge of the lesion offers the best information for the pathologist. A biopsy of the center of the lesion (choice A) deprives the pathologist of all the clues that are found at the interface between the tumor and the normal skin, and in large lesions it runs the risk of sampling necrotic tumor that has outgrown its blood supply. A wide excision before pathologic diagnosis (choice C) risks doing too much (a basal cell cancer needs only 1 or 2 mm of margins) or too little (a melanoma should have at least 2 cm). Radiation therapy (choice D) is a viable therapeutic choice for squamous cell carcinoma, but not before a diagnosis has been established. Here, we are expecting a basal cell carcinoma, thus this course of action would be even less appropriate. Scrapings and cultures (choice E) assume an infectious process, ignoring the strong clinical suspicion of a tumor in this case. Clinical Pearls Basal cell carcinoma (BCC), the most common malignancy of Caucasians, is a skin cancer that arises from the basal layer of the epidermis. The incidence is increasing worldwide and is linked to ultraviolet light exposure as a child. The tumors have a low metastatic potential, but can be locally destructive. Most BCC occurs on the face and head and diagnosis of any suspicious lesion needs to be confirmed histologically with biopsy. Biopsies should always include some of the suspicious tissue as well as some of the normal-appearing surrounding skin.

A 35-year-old woman has a pigmented lesion removed from her palm for cosmetic reasons. The lesion is light brown in color, consisting of a 2 mm very slightly elevated macule with well demarcated edges. The adjacent skin has normal color. Microscopic examination of the lesion demonstrates clustering of pigmented melanocytes at the epidermodermal junction. No clusters of melanocytes are seen in the dermis. Which of the following is the most likely diagnosis? A. Compound nevus B. Junctional nevus C. Halo nevus D. Intradermal nevus E. Lentigo

The correct answer is B. The patient has a junctional nevus, which is one type of mole. Junctional nevi may be light brown to nearly black, and range in size from 1 to 10 mm. They may be either flat or very slightly raised. It is thought that almost all melanocytic nevi may begin as junctional nevi. In adulthood, the lesions are seen most frequently on the palms, soles, and genitalia. Clustering of melanocytes at the dermal-epidermal junction is characteristic microscopically. These lesions are very common, and removal is usually undertaken only when the mole has been exhibiting color changes; is irritated; or is cosmetically a problem. Compound nevus (choice A) is characterized by clusters of melanocytes both at the epidermodermal junction and within the dermis. A halo nevus (choice C) is a compound or intradermal nevus surrounded by an area of intense inflammation. The inflammation destroys the melanocytes of the immediately adjacent skin, producing a pale halo around the nevus. These typically are benign lesions. Intradermal nevus (choice D) is characterized by clusters of melanocytes with the dermis but not at the junction. Lentigo (choice E) clinically closely resembles a junctional nevus, but microscopically there is an increased number of melanocytes occurring as isolated cells, rather than in nests, in the lower epidermis. Clinical Pearls Acral melanocytic nevi are nevi that occur on the palms and soles of the feet and are more common in patients with darker skin. Acral melanocytic nevi are usually junctional or compound nevi and are generally brown on dark brown. Lesions with marked asymmetry, mottled pigmentation, or lesions ge6 mm should be referred to a dermatologic healthcare provider for evaluation and management.

A 4-year-old girl is brought to the healthcare provider because of a crusted honey-colored erythema resulting from rupture of tiny vesicles and pustules. Her temperature is 37.7° C (102° F). Skin lesions are distributed over the face and extremities. Physical examination reveals enlargement of lymph nodes in the cervical and axillary regions. Which of the following is the most frequent pathogen of this skin infection? A. Human herpesvirus 6 B. Staphylococcus aureus C. Streptococcus pyogenes D. Propionibacterium acnes E. Trichophyton fungi

The correct answer is B. The presentation is consistent with impetigo. Impetigo may be of 2 forms, nonbullous and bullous. Nonbullous impetigo (more frequent) develops by colonization of traumatized skin by pyogenic bacteria and manifests with the characteristic vesiculopustular eruption that results in honey-colored crusts. Bullous impetigo is due to a strain of S. aureus that produces exfoliatin, a toxin causing intraepidermal cleavage. Bullous impetigo manifests with large flaccid bullae that rupture easily. Currently, S. aureus is the etiologic agent of most cases of nonbullous impetigo, while until 2 decades ago S. pyogenes (choice C) accounted for the majority of cases. This change has important therapeutic implications. S. pyogenes is sensitive to penicillin, while S. aureus is not. Thus, the most appropriate treatment of both forms of impetigo is based on antibiotics effective against S. aureus. Treatment options include systemic erythromycin, first-generation cephalosporins, and topical mupirocin. Mupirocin is a fermentation product deriving from Pseudomonas fluorescens. It acts by inhibiting bacterial protein synthesis. Human herpesvirus 6 (choice A) is suspected to be the etiologic agent of roseola. This self-limited dermatosis is characterized by scaly pink macules that affect young children under the age of two. High fever typically predates the rash. Propionibacterium acnes (choice D) is thought to play an important role in the pathogenesis of acne. Colonization by this bacterium leads to release of pro-inflammatory and chemotactic agents that induce inflammation of the pilosebaceous unit. Trychophyton fungi (choice E) are the most common pathogenic dermatophytes causing tinea corporis, due to superficial invasion of the stratum corneum by fungal organisms. Clinical Pearls Impetigo is a highly contagious superficial bacterial infection that usually involves the face and extremities. Lesions usually start as papules and then transform into vesicles or pustules and ultimately crust over. The most common cause of impetigo is Staph aureus, but some cases are caused by beta-hemolytic streptococci (primarily group A). Some cases are caused by both staph and strep. Cases of impetigo can occasionally be followed by poststreptococcal glomerulonephritis or rheumatic fever. First-line treatment for impetigo is topical mupirocin applied to the affected areas 3 times daily. Oral antibiotic treatment with dicloxacillin, cephalexin, or clindamycin is recommended for bullous impetigo. If patients are high risk for MRSA or have a beta-lactam allergy, treatment with clindamycin or linezolid (Zyvox) may be appropriate. All patients should be instructed on the importance of handwashing to prevent the spread of impetigo.

A 62-year-old woman who recently moved to the area comes to a new healthcare provider for her semiannual skin check. She has a history of psoriasis and multiple precancerous skin lesions. She also has diabetes and hypertension that are treated with appropriate medications and are under good control. There is no family history of skin diseases or skin cancers. She grew up in Florida and spent lots of time unprotected in the sun in her youth. When the healthcare provider inquires how she treats her psoriasis, she says that she really does not because it is a single spot on her back that did not respond to the steroid cream she was prescribed and never really bothered her much but seems to be getting a little larger with the years. Physical examination shows extensive actinic damage on sun-exposed areas of the upper body and legs with multiple solar lentigines and telangiectasias. On her mid-back there is a single well-demarcated, erythematous, scaly patch of approximately 20 cm2 with multiple crusted areas within it and an irregular, scalloped border. Which of the following is the most likely diagnosis? A. Allergic contact dermatitis B. Bowen disease C. Keratoacanthoma D. Melanoma E. Psoriasis

The correct answer is B. This patient has Bowen disease, or squamous cell carcinoma in situ, a specific type of squamous cell carcinoma that has a long, indolent course but may become invasive and even metastasize after many years of superficial growth. Clinically, it presents as an erythematous, scaly patch with irregular borders that slowly enlarges over many years. During that time, minute areas of erosion develop and appear as crusts. It may have an eczematous (scaly and crusted) or psoriasiform (thick, whitish scale) appearance and is commonly initially misjudged to be an inflammatory disorder. However, it does not respond to topical antiinflammatory treatment. The cumulative effects of ultraviolet radiation are a significant risk factor. Surgical excision, cryosurgery, curettage with electrodesiccation, and topical 5-fluorouracil are treatment options after histopathologic examination has confirmed the diagnosis. It would be very unlikely that a single lesion of allergic contact dermatitis (choice A) persists for a long period of time without waxing and waning, responding to topical treatment, or causing some kind of symptoms, such as itching or stinging. In addition to that, allergic contact dermatitis is usually ill defined because the inflammatory process spreads beyond the area of contact with the allergen. In the acute phase, erythematous and edematous vesiculated plaques are characteristic. Long-standing, chronic lesions become thickened and lichenified with accentuated skin markings and hyperpigmentation secondary to trauma-induced necrosis of keratinocytes and their subsequent elimination into the dermis. Keratoacanthoma (choice C) erupts abruptly, grows to its maximum size within weeks or months, and then, most commonly, undergoes spontaneous resolution resulting in an atrophic scar. The classic, fully developed lesion is an indurated, telangiectatic nodule that harbors a central crater depression filled with adherent, keratotic material. It may be from a few millimeters to a few centimeters in diameter. When the central keratotic core is removed, the base bleeds. Melanoma (choice D) typically presents as an asymmetric, brown to black, smooth patch, papule or plaque with irregular scalloped borders and variegation in color throughout the lesion. More advanced, nodular melanoma is typically a bluish-black nodule that bleeds easily and has an adjacent variegated hyperpigmented or erythematous component from which it stems. Skin markings are absent on the surface of the lesion, and there may be signs of regression with white or gray areas within the hyperpigmented component. Psoriasis (choice E) is an inflammatory disorder that may be present at birth or appear later in life. In approximately 30% of patients there is a positive family history. Clinically, well-demarcated, erythematous papules and plaques covered with nonadherent, silvery-white scale are typical. Sites of predilection include the scalp, ear canals, elbows, knees, umbilicus, and nails. As with any chronic inflammatory skin disease, multiple lesions are a rule, and the course is one of remissions and exacerbations. Clinical Pearls: Bowen's disease, also known as cutaneous SCC in situ, is characterized by an asymptomatic scaly, well-demarcated patch/plaque that grows slowly over a course of years. The lesion can be erythematous, but can also be pigmented or skin-colored. Because of its long, indolent course, SCC can be confused with psoriasis so consider this as a diagnosis if patients fail to respond to treatment of psoriasis.

A 24-year-old woman comes to the emergency department because of an acute onset of abdominal cramping and hives in the previous 12 hours. She states that 2 days ago she experienced a bad sore throat and took some leftover trimethoprim-sulfamethoxazole (Bactrim), an over-the-counter cold medicine containing acetaminophen, diphenhydramine (Benadryl), and pseudoephedrine, and several doses of aspirin. She has a history of asthma that is exacerbated with upper respiratory infections, so she tried to treat herself before the asthma was triggered. Otherwise, her medical history is significant for allergy to peanuts and chocolate, which she meticulously avoids. Review of systems reveals a generalized itch that is unbearable, and abdominal cramping of moderate degree. The patient denies difficulty swallowing or breathing. On physical examination she is in mild distress. Her blood pressure is 105/65 mmHg, pulse 90/min, and respirations 24/min. Body is 36° C (96.8° F). Inspection reveals multiple 1- to 10-cm pink and white urticarial papules and plaques on the trunk and proximal extremities. Her eyelids are edematous and the lower lip appears to be slightly edematous as well. Which of the following is the most likely cause of the skin lesions? A. Acetaminophen B. Aspirin C. Diphenhydramine (Benadryl) D. Pseudoephedrine E. Trimethoprim-sulfamethoxazole (Bactrim)

The correct answer is B. This patient has urticaria and angioedema induced by a hypersensitivity reaction, most likely to aspirin. Drugs are probably the most frequent cause of urticaria and angioedema, penicillin and related antibiotics being the most frequent offenders. Salicylates, aspirin, opiates, nonsteroidal anti-inflammatory drugs, contrast media, and angiotensin-converting enzyme inhibitors are common causes of urticaria. Acetaminophen (choice A) is a cyclooxygenase inhibitor that works well for the treatment of fever and noninflammatory pain. It is relatively safe to use, has few side effects, and does not interact with many other medications in a significant way. The only persons who should avoid taking acetaminophen are those with chronic liver problems or high alcohol intake where hepatic toxicity may be a concern. It is not a known cause of acute urticaria. Diphenhydramine (choice C) is a first-generation H1 antihistamine. Systemic adverse effects can be divided into those affecting the central nervous system (sedation, irritability, impaired cognitive function, increased appetite), gastrointestinal (constipation, dry mouth), genitourinary (dysuria, erectile dysfunction), cardiac (tachycardia, dysrhythmia), and other (blurred vision). It is used in the treatment of mast cell-mediated urticaria and angioedema, but in doses considerably higher than what is available in an over-the-counter cold medicine. Pseudoephedrine (choice D) is a decongestant commonly used for the treatment of allergic or infectious rhinitis. Its main adverse effect is xerostomia. It is not a common cause of urticaria and angioedema. Trimethoprim-sulfamethoxazole (choice E) most commonly shows adverse reaction in the gastrointestinal system and the skin. Maculopapular exanthemata are seen in patients with AIDS. Otherwise, pustular reactions, Sweet syndrome, Stevens-Johnson syndrome, and toxic epidermal necrolysis have been reported. Central nervous system effects include tremor, fatigue, and headache. It is not a common cause of urticaria in immunocompetent patients. Clinical Pearls Angioedema is a localized swelling of skin or mucous membranes that is usually self-limited. It can occur with urticaria or anaphylaxis but can also be an isolated finding. The underlying pathophysiology of angioedema is loss of vascular integrity secondary to inflammatory mediators, which lead to leakage of fluid into the interstitial spaces. Causes include NSAIDs (ASA or ibuprofen), which inhibit the COX-1 enzyme, responsible for mediating the generation of prostaglandins from arachidonic acid in mast cells and other leukocytes. NSAIDs can therefore result in production of pro-inflammatory mediators, which can lead to angioedema in certain individuals. Once angioedema occurs, medications from that class of medications should be strictly avoided due to the potential for anaphylaxis upon further exposure.

A 32-year-old executive comes to the healthcare provider complaining of an itchy rash that developed on his thighs over the previous several weeks. It is very disturbing and causes embarrassing situations because he spends a lot of his time in meetings and cannot suppress the need to scratch throughout the day. It calms down somewhat overnight, only to return full force the next day. He has no other medical problems and has not taken any medication recently. He is not aware of any allergies to medication but does note that he cannot wear a metal watch because he develops eczema on the wrist. Physical examination shows poorly circumscribed erythematous, scaly and crusted, moist plaques with multiple excoriations on both upper lateral thighs. On closer inspection there are tiny, translucent vesicles within the plaques. As the patient finishes dressing the healthcare provider notices that the patient places his car keys into one trouser pocket and some loose change into the other. This condition is most likely the result of which of the following mechanisms? A. Cytotoxic hypersensitivity B. Delayed hypersensitivity C. Immediate hypersensitivity D. Immune complex-mediated hypersensitivity E. Trauma-induced

The correct answer is B. This patient most likely has allergic contact dermatitis to metal, most likely caused by hypersensitivity to nickel. Allergic contact dermatitis is caused by a delayed-type hypersensitivity reaction, an immunologically specific reaction, which takes some hours to reach a maximum and occurs in the absence of demonstrable antibody in serum. The reaction is mediated by T lymphocytes with receptors for specific antigen on their membrane. There are usually few cells with antigen specificity, but when activated after exposure to antigen, they attract many other cells of different types, which contribute to the reaction. Our patient had developed erythema, edema, and vesiculation at the site of contact with metal objects carried in his pockets, and he already knows that he develops "eczema" when he wears a metal watch. This all points to a metal allergy, and of all metals, nickel is the most common offender. Cytotoxic hypersensitivity (choice A) results in cell damage due to antibody that is specific for an antigen that has become intimately absorbed or incorporated in the structure of a cell. An example is the lysis of red blood cells that have absorbed bacterial polysaccharide on their surface, when treated with a complement-fixing antibody specific for the bacterial antigen. Other examples of cytotoxic changes are those resulting from antibody or "primed" leukocytes reacting with skin antigens, as may occur in autoallergy and in rejection of homografts. The skin may also be affected secondarily to a cytotoxic reaction occurring in the blood vessels: thus purpura follows the destruction of platelets and probably endothelium when antibody to a drug reacts with the drug adsorbed on cells. This is seen with platelets in thrombocytopenic purpura and red cells in hemolytic anemia. Immediate hypersensitivity (choice C), or anaphylaxis, occurs when mast cells or basophils in tissue, passively sensitized with antibody, are exposed to specific antigen. The immediate weal and flare of a prick test response of an anaphylactically sensitized person is a good example of an anaphylactic or immediate-type hypersensitivity reaction. In humans, the IgE antibody is that of anaphylaxis. Persons with a predisposition to form IgE antibodies and with an increased susceptibility to IgE-mediated (hay fever) or IgE-associated (atopic dermatitis) disorders are characterized as being atopic. Urticaria and angioedema are classical signs of cutaneous anaphylaxis. Immune complex-mediated hypersensitivity (choice D) is classically represented by serum sickness. Soluble immune complexes result from the union of antibody with antigen in the blood and are dispersed throughout the entire body. Formation of complexes is a normal process in the body to remove foreign material. It is only when there is insufficient antibody and the complexes penetrate vessel walls that disease results. The disorders induced are disseminated or local necrotizing vasculitis, glomerulonephritis, and arthritis. Trauma-induced (choice E) lesions are usually well-demarcated skin lesions devoid of a significant inflammatory component (unless produced by application of strong irritants such as acid or alkalis). They may present as lichenified papules or plaques (from persistent scratching), keratotic lesions (from picking), or ulcerations (from application of strong physical or chemical irritants). Their distribution reflects the offending agent and is usually random. Clinical Pearls: Allergic contact dermatitis (ACD) refers to an inflammatory skin disease characterized by pruritic, eczematous lesion that are a delayed type hypersensitivity reaction mediated by T cells that occur after the skin is contacted by an offending chemical in patients who have been previously sensitized to the chemical. ACD is the most common occupational skin disease and is commonly seen in healthcare workers, chemical industry workers, beauticians and hairdressers, machinists, and construction workers. Common causes include metals, glues, plastics, rubber, fragrances, topical antibiotics, preservatives, and chemicals used in hair-care and cosmetic products.

Which of the following types of malignant melanoma most commonly occurs in darkly pigmented populations? A. Superficial spreading B. Lentigo maligna melanoma C. Acral lentiginous D. Nodular

The correct answer is C. Acral lentiginous is the most common malignant melanoma type in darkly pigmented patients. Locations include palms, soles, mucosal surfaces, nail beds, and mucocutaneous junctions. Superficial spreading (choice A), lentigo maligna melanoma (choice B), and nodular (choice D) subtypes do not occur most commonly in darkly pigmented populations.

Which of the following types of malignant melanoma most commonly occurs on the palms, soles, and mucosal surfaces? A. Superficial spreading B. Lentigo maligna melanoma C. Acral lentiginous D. Nodular

The correct answer is C. Acral lentiginous is the most common malignant melanoma type in darkly pigmented patients. Locations include palms, soles, mucosal surfaces, nail beds, and mucocutaneous junctions. Superficial spreading (choice A), lentigo maligna melanoma (choice B), and nodular (choice D) subtypes do not occur most commonly on the palms, soles, and mucosal surfaces.

A patient presents to the office with several skin lesions that are identified as actinic keratosis. These skin lesions are premalignant lesions for which of the following types of cancer? A. Malignant melanoma B. Basal cell carcinoma C. Squamous cell carcinoma D. Kaposi sarcoma

The correct answer is C. Actinic keratosis is a premalignant squamous cell carcinoma. Actinic keratosis is not premalignant lesion for malignant melanoma (choice A), basal cell carcinoma (choice B), or Kaposi sarcoma (choice D).

A patient presents to the office with concerns about a skin lesion on her face. The lesion is a pearly, translucent, smooth papule with rolled edges and surface telangiectasia. Which of the following is the most likely diagnosis? A. Normal finding B. Melanoma C. Basal cell carcinoma D. Squamous cell carcinoma

The correct answer is C. Basal cell skin lesions are classically described as pearly, translucent, smooth papules with rolled edges and surface telangiectasia. Normal findings (choice A), melanoma (choice B), and squamous cell carcinoma (choice D) do not appear as pearly, translucent, smooth papules with rolled edges and surface telangiectasias.

A 6-year-old boy is brought to the pediatric healthcare provider because of a 3-day history of skin lesions. On physical examination, he has multiple yellow, crusted erosions below the nares and on the cheeks, chin, and upper extremities. The rest of the examination is normal. Which of the following is the most appropriate treatment for this condition? A. Oral acyclovir B. Oral amoxicillin C. Oral cephalexin D. Topical ketoconazole (Nizoral Topical) E. Topical 2% hydrocortisone

The correct answer is C. Bullous impetigo (staphylococcal impetigo) is caused by an epidermolytic toxin produced at the site of infection, most commonly by staphylococci of phage group II. The toxin causes intraepidermal cleavage below or within the stratum granulosum. Bullous impetigo is most common in infants and children. It typically occurs on the face, but it may infect any body surface. There may be a few lesions localized in one area, or the lesions may be numerous and widely scattered. One or more vesicles enlarge rapidly to form bullae in which the contents turn from clear to cloudy. The center of the thin-roofed bulla collapses, and a thin, flat, honey-colored crust may appear in the center with a bright red, inflamed, moist base that oozes serum. In most cases, a tinea-like scaling border replaces the fluid-filled rim as the round lesions enlarge and become contiguous with the others. The border dries and forms a crust. The lesions have little or no surrounding erythema. Regional lymphadenitis is uncommon with pure staphylococcal impetigo. There is some evidence that the responsible staphylococci colonize the nose and then spread to normal skin prior to infection. Serious secondary infections (e.g., osteomyelitis, septic arthritis, and pneumonia) may follow seemingly innocuous superficial infections in infants. The drug of choice for impetigo is cephalexin (Keflex). Cloxacillin, dicloxacillin, and azithromycin are alternatives. Because some cases of impetigo are due to a mixed staphylococcal/streptococcal infection penicillin and amoxicillin (choice B) provide inadequate treatment. Oral acyclovir (choice A) is used to treat herpes simplex virus infection. Topical ketoconazole (choice D) is used to treat fungal infection of the skin. Topical 2% hydrocortisone (choice E) is ineffective against impetigo, which is caused by a bacterial infection. Clinical Pearls Impetigo is a highly contagious superficial bacterial infection that usually involves the face and extremities. Lesions usually start as papules and then transform into vesicles or pustules that ultimately crust over. The most common cause of impetigo is Staph aureus, but some cases are caused by beta-hemolytic streptococci (primarily group A). Some cases are caused by both staph and strep. Cases of impetigo can occasionally be followed by poststreptococcal glomerulonephritis or rheumatic fever. First line treatment for impetigo is topical mupirocin applied to the affected areas 3 times daily. Oral antibiotic treatment with dicloxacillin, cephalexin, or clindamycin is recommended for bullous impetigo. If patients are high risk for MRSA or have a beta-lactam allergy, treatment with clindamycin or linezolid (Zyvox) is appropriate. All patients should be instructed on the importance of handwashing to prevent the spread of impetigo.

Abnormal immune reactions play a crucial role in the pathogenesis of many dermatologic conditions. Which of the following disorders is mediated by a type IV hypersensitivity reaction? A. Acne vulgaris B. Bullous pemphigoid C. Contact dermatitis D. Dermatitis herpetiformis E. Discoid lupus erythematosus

The correct answer is C. Contact dermatitis is a form of eczematous dermatitis, characterized clinically by itchy vesicular eruption and histologically by epidermal spongiosis and chronic dermal inflammation. Contact dermatitis is initiated by hypersensitivity to an antigen that is taken up by Langerhans cells. These antigen-presenting cells process antigens and present them to naive CD4 positive (helper) T lymphocytes. Antigen re-exposure leads to recruitment of helper T cells and release of cytokines that mediate the inflammatory response. The pathogenesis of acne vulgaris (choice A) is related to endocrine, familial, and environmental factors. Bacterial lipases from Propionibacterium acnes may play a role. Bullous pemphigoid (choice B) is mediated by different types of autoantibodies that react with components of desmosomes or hemidesmosomes, disrupting intercellular junctions and resulting in bulla formation. Thus, the immune mechanism of these conditions can be regarded as type II hypersensitivity reactions. Dermatitis herpetiformis (choice D) is a blistering disease that develops in individuals allergic to gliadin, a gluten component. IgA and IgG antibodies react with gliadin and deposit in the tips of dermal papillae, leading to inflammation and bulla formation. Type III hypersensitivity therefore appears to be the prevalent immunologic mechanism. Discoid lupus erythematosus (choice E) is the localized cutaneous form of systemic lupus erythematosus. Skin lesions are associated with immune complex deposition along the dermal-epidermal junction. This is a form of type III hypersensitivity reaction. Clinical Pearls Type IV hypersensitivity reactions involve the activation T cells and other cells (macrophages, eosinophils, neutrophils) and do not activate antibodies. Allergic contact dermatitis (ACD) refers to an inflammatory skin disease characterized by pruritic, eczematous lesion that are a delayed type hypersensitivity reaction mediated by T cells that occur after the skin is contacted by an offending chemical in patients who have been previously sensitized to the chemical. ACD is the most common occupational skin disease and is commonly seen in healthcare workers, chemical industry workers, beauticians and hairdressers, machinists, and construction workers. Common causes include metals, glues, plastics, rubber, fragrances, topical antibiotics, preservatives, and chemicals used in hair-care and cosmetic products.

A 4-year-old boy has a 5-day history of fever and increased irritability. His temperature is 40.2° C (104.3° F), blood pressure is 98/68 mm Hg, pulse is 112/min, and respirations are 24/min. On physical examination, he is noted to have bilateral cervical lymphadenopathy, cracked lips, and bilateral conjunctival injection. Examination of his tongue reveals hypertrophy of the papillae in a bright red background. His palms and soles are erythematous. There is a polymorphous macular rash generalized on his body. Which of the following is the most appropriate pharmacotherapy? A. Amoxicillin B. Aspirin and corticosteroid C. Aspirin and IV immune globulin D. Corticosteroid E. IV immune globulin

The correct answer is C. Kawasaki disease is a systemic vasculitis of unknown origin that remains a leading cause of acquired heart disease in infants and children. It is a multisystemic disease also known as mucocutaneous lymph node syndrome. Clinical and echocardiographic features remain the basis for diagnosis. An (unidentified) infectious origin and a T-cell immune activation play a prominent role in disease pathogenesis. Tumor necrosis factor alpha (TNF-alpha) receptor levels correlate with the degree of vascular damage and likelihood of coronary artery aneurysm formation. The cardiovascular complications account for most of the morbidity and mortality. Fever, bilateral non-exudative conjunctivitis, mucous membrane changes (injected pharynx, cracked lips, or strawberry tongue), extremity changes (edema, desquamation, erythema, or rash), and cervical adenopathy are common at presentation. The acute manifestations include myocarditis, valvular insufficiency, arrhythmias, pericardial effusion, and congestive heart failure with coronary abnormalities; these develop in 15% to 25% of patients. Leukocytosis and an elevated C-reactive protein are associated with the development of coronary artery aneurysms. Treatment includes aspirin at 80-100 mg/kg/day in divided doses and IV immune globulin in high doses. Corticosteroids should be avoided because of the potential link with increasing the likelihood of coronary aneurysm development. Amoxicillin (choice A) treats bacterial infections including strep pharyngitis, acute otitis media, sinusitis, H. pylori infection, Lyme disease, and acute salmonella. Aspirin and corticosteroids (choice B) are not the proper combination of therapy for Kawasaki disease. Corticosteroids (choice D) may help with other forms of vasculitis, but it is not specific enough to treat Kawasaki disease. IV immune globulin (choice E) is not as efficacious when used alone for managing Kawasaki disease as when used in combination with aspirin. Clinical Pearls Kawasaki disease (KD, also called mucocutaneous lymph node syndrome) is a common vasculitic disease of childhood. There are no specific diagnostic studies to confirm the diagnosis; diagnostic criteria include fever along with signs of mucocutaneous inflammation.. Treatment is a single dose of IVIG combined with aspirin. This condition may be complicated by coronary artery thrombosis secondary to inflamed, aneurysmal vessels.

A 32-year-old African American woman comes to the clinic complaining of "bumps" that developed on her earlobes in the past 3 months, are very itchy, and seem to be enlarging. She had her ears pierced 6 months earlier and had a lot of problems with infection of the pierced openings, which healed only after she had taken a course of oral antibiotics. She is aware that she heals poorly and develops unsightly scars but hoped that such a small procedure as ear piercing would not be a problem. She does not have any significant medical problems and does not take any medication. On physical examination, there are bilateral rubbery firm, skin-colored nodules on her posterior earlobes adjacent to the sites of the piercing. They are somewhat mobile related to deeper tissue and are tender to palpation. On her chest and shoulders there are several round and linear, skin-colored, firm scars that have pseudopodia extending into the surrounding skin, giving them a starfish-like appearance. Which of the following is the most appropriate treatment for the lesions on her ears? A. Compressive earrings B. Immediate surgical excision C. Intralesional corticosteroid injection D. Systemic corticosteroids E. Topical corticosteroid

The correct answer is C. Keloids are a type of exuberant scar-tissue formation that tends to run in families and has a higher incidence in African Americans. This patient has keloids that are in the proliferating stage and are best treated with intralesional corticosteroid injections until they stop active growth, reduce in size and stop itching, at which time they may be surgically excised with greater ease. The injections are usually performed with 0.5 to 1 mL of triamcinolone solution 20 to 40 mg/ml every 4 to 6 weeks. It may take multiple injections for months before a satisfactory result is achieved. Prophylaxis includes using atraumatic surgical equipment, injecting the wound with corticosteroids intraoperatively, and applying compression during convalescence. Unfortunately, even if all precautions are taken, the result may not be satisfactory and multiple injections may be required. Compressive earrings (choice A) are used to prevent keloids from recurring at the site of their excision on the earlobes. They are effective only if used before the keloid has formed. Applying compression to already existing keloids will not improve their appearance and may be extremely uncomfortable or frankly painful for the patient. Various compression devices are available for prevention of keloids on other body sites besides the earlobes. Immediate surgical excision (choice B) is not recommended because recurrence is practically guaranteed unless the abnormal proliferative state is curtailed before surgery. With intralesional corticosteroid injections, over several months the keloids will stop growing, soften, and even shrink in size. Once that occurs, surgery may performed with less risk of recurrence. Systemic corticosteroids (choice D) are not effective in the treatment of keloids. The tissue levels attained with systemic therapy are not high enough to arrest the extreme fibroblast activity. Intralesional injection provides very high local tissue levels and is therefore the treatment of choice. Topical corticosteroids (choice E) have no effect on keloids. High-potency topical steroids may have some effect when used early in the postsurgical period. Massaging the new scar twice a day with a fluorinated corticosteroid cream is recommended. Clinical Pearls Keloids differ from hypertrophic scars in that they extend beyond the borders of the causative wound with linear projections and pseudotumor-like growth. They also tend to be symptomatic (tender, pruritic) early in their course. Treatment is intralesional corticosteroid injections. Excision is discouraged because the chance of recurrence is very high.

A previously healthy 27-year-old woman, gravida 2, para 2 is 4 days status post-cesarean section and develops a temperature to 100.7° F (38.2° C). She had her cesarean section when she went into unstoppable preterm labor with a breech fetus. She had an uncomplicated postoperative course until this temperature elevation. Her pulse is 100/min, blood pressure 110/70 mm Hg, and respirations 16/min. There is discoloration and cyanosis around the incision, with a cloudy, dishwater like discharge. The area around the incision is completely numb. There is no uterine tenderness on bimanual examination. Which of the following is of the most concern? A. Endometritis B. Mastitis C. Necrotizing fasciitis D. Preeclampsia E. Wound infection

The correct answer is C. Necrotizing fasciitis is a rare but potentially fatal complication of abdominal wound infection. It typically occurs in patients who are immunocompromised or who have diabetes or cancer. It is a clinical diagnosis that is characterized by discoloration and cyanosis around the incision with numbness of the area. It can be polymicrobial in nature, but anaerobes are frequently involved. It is considered to be a potentially fatal condition and aggressive treatment with broad-spectrum antibiotics and surgical debridement is essential. Endometritis (choice A) is characterized by abdominal pain, malaise, foul-smelling lochia, temperature elevation, and uterine tenderness on bimanual examination. This patient does not have uterine tenderness on bimanual examination and her disease process appears focused around the incision site. Therefore, endometritis would not be the process of most concern in this patient. Mastitis (choice B) is an infection of the breast that is characterized by breast pain, elevated temperature, erythema and edema of the breast. This patient's process is not involving the breast; therefore, mastitis would not be of concern here. Preeclampsia (choice D) is characterized by hypertension, edema, and proteinuria. The cure for preeclampsia is delivery of the fetus. This patient has no findings concerning for preeclampsia. This patient has features to her presentation that suggest more than a simple wound infection (choice E). The discoloration of the wound edges and cyanosis, along with the loss of sensation, point toward the more worrisome process of necrotizing fasciitis. Clinical Pearls Necrotizing fasciitis is a deep infection of the subcutaneous tissue that leads to progressive destruction of the underlying tissues, such as the subcutaneous fat and fascia, but may spare the skin. There are 2 types of necrotizing fasciitis: Type I necrotizing fasciitis represents a polymicrobial infection caused by both aerobic and anaerobic bacteria and is usually seen in the setting of recent surgery, diabetes, and peripheral vascular disease. Type II necrotizing fasciitis represents a monomicrobial infection caused by group A streptococcus (GAS or Strep. pyogenes). Recently there has also been evidence of necrotizing fasciitis caused by MRSA. Post-surgical necrotizing fasciitis is characterized by copious dishwater-like drainage, dusky and friable subcutaneous tissue, and a pale, devitalized fascia. Post-surgical necrotizing fasciitis is a potentially fatal wound infection and should be considered a surgical emergency.

An 8-year-old boy is brought to the healthcare provider for a routine health maintenance visit. His mother states that he has had difficulty reading and concentrating in his second-grade class. On examination, there are seven pigmented cutaneous lesions ranging from light to dark brown on his body, as well as 2 small, soft masses above his orbit, are seen. He also has numerous freckles in the axillae. His mother has similar lesions to the ones that he has throughout his body. Which of the following is the most likely diagnosis? A. Congenital hypothyroidism B. Marfan syndrome C. Neurofibromatosis D. Osteogenesis imperfecta E. Tuberous sclerosis

The correct answer is C. Neurofibromatosis (NF) is a multisystem genetic disorder. The features of this condition are ≥6 café-au-lait spots, ≥2 neurofibromas, axillary freckling, optic gliomas, iris hamartomas (Lisch nodules), and osseous lesions. There is almost always a first-degree relative with neurofibromatosis. Congenital hypothyroidism (choice A) is associated with poor feeding, an enlarged fontanelle, an enlarged tongue, and an umbilical hernia in the neonatal period. It should not go undetected until 8 years of age. Marfan syndrome (choice B) is a connective tissue disorder characterized by long fingers, hypermobile joints, subluxation of the lenses, pectus carinatum, and aortic aneurysms. Osteogenesis imperfecta (choice D) is a rare connective tissue disease characterized by recurrent fractures, blue sclera, thin skin, and hyperextensibility of ligaments. Tuberous sclerosis (choice E) is associated with facial angiofibromas (adenoma sebaceum), retinal hamartomas, seizures, and mental retardation. Seizures are the most common presenting symptom. It is an autosomal-dominant inherited disease. Clinical Pearls Neurofibromatosis (NF) describes 2 distinct genetic conditions than can lead to neurocutaneous tumors. Von Recklinghausen's (NF type 1), an autosomal dominant disease, is the more common disease, affecting 1 in 2,800 people. Half of all cases of NF type 1 are familial. Café-au-lait spots are the hallmark of type 1 and the lesions are usually multiple along with multiple cutaneous neurofibromas. The National Institutes of Health (NIH) has developed diagnostic criteria for NF type 1 and at least 2 of the criteria need to be present to make the diagnosis: ≥2 cutaneous neurofibromas of any type - benign tumors with a mixture of Schwann cells, fibroblasts, and mast cells; or a single plexiform neurofibroma ≥6 café-au-lait spots - flat uniformly hyperpigmented macules - measuring the longest diameter of greater than 5 mm in prepubertal patients and greater than 15 mm in postpubertal patients. Axillary or inguinal freckling. Optic glioma ≥2 Lisch nodules (hamartomas of the iris) Bony lesions such as sphenoid dysplasia or thinning of the long bone cortex with or without pseudoarthrosis First-degree relative with NF type 1 by the same criteria.

A 5-year-old boy, hospitalized for cellulitis, exhibits global denudation of the skin resulting from splitting of the epidermis at the stratum granulosum. Which of the following is the most likely diagnosis? A. Impetigo B. Melasma C. Scalded skin syndrome D. Tinea corporis E. Vitiligo

The correct answer is C. Scalded skin syndrome is a pediatric condition caused by an exfoliative toxin produced by Staphylococcus aureus (which is a common cause of cellulitis). The toxin splits the epidermis at the level of the stratum granulosum, causing global denudation of the skin. Scalded skin syndrome is not associated with suntanning or sunburn. Impetigo (choice A) is a superficial skin infection, usually caused by group A beta-hemolytic streptococci or staphylococci. Classic clues are eroded pustules covered by honey-colored crusts. Impetigo may lead to poststreptococcal glomerulonephritis, a sign of which is red cell casts in the urine. Melasma (choice B) consists of irregular patches of hyperpigmentation on the face. It most commonly appears during pregnancy and may not completely regress. Tinea corporis (choice D) is a fungal infection. It is also known as ringworm because it presents as an expanding round lesion with an erythematous circinate border. Vitiligo (choice E) is characterized by irregular patchy depigmentation of the skin that exhibits melanocyte deficiency microscopically. It is possibly autoimmune in origin and may be related to stress. It is not associated with sun exposure and is a chronic condition. Clinical Pearls Staphylococcal scalded skin syndrome (SSSS) is caused by epidermolytic toxins produced by certain strains of staphylococci. This toxin is distributed systemically and results in dissolution of keratinocyte attachments in only the upper layer of the epidermis (stratum granulosum). SSSS usually affects newborns and children. Adults are less commonly affected because improved renal function allows for clearance of the toxins from the body. This condition needs to be differentiated from Stevens-Johnson syndrome (SJS) by its sparing of the mucus membranes. Antibiotic treatment with antistaphylococcal antibiotics or vancomycin (MRSA) along with the use of emollients is the treatment of choice.

Which of the following is the most likely site for basal cell skin carcinoma to occur? A. Lower legs B. Forearms C. Face D. Back E. Neck

The correct answer is C. The face is the most common site for basal cell carcinoma, as this part of the body typically has the most sun exposure. The lower legs (choice A), forearms (choice B), back (choice D), and neck (choice E) are not the most common sites for basal cell carcinoma because they have less sun exposure.

An elderly patient complains to a healthcare provider of sores in the mouth and on the skin. These lesions have developed over about a month. Physical examination demonstrates multiple painful erosions on the oral mucosa and tongue. Raw areas with crusting on the skin are seen on the face and trunk. The patient states that the skin lesions had started as blisters that had quickly broken. While the involved areas were painful, no itchiness had been experienced. Careful examination of the edge of the skin lesions demonstrates a few flaccid bullae. Rubbing of the skin near an affected area easily detaches the superficial part of the epidermis from the underlying skin. No target-like lesions are seen. Which of the following is the most likely diagnosis? A. Bullous pemphigoid B. Dermatitis herpetiformis C. Pemphigus vulgaris D. Stevens-Johnson syndrome E. Toxic epidermal necrolysis

The correct answer is C. The patient has pemphigus vulgaris, which is an uncommon autoimmune skin disorder characterized by blistering and erosions involving the mucous membranes and skin. The autoimmune attack is on the junctions between epithelial cells in the epidermis. The blisters occur high in the epithelium and can rupture easily, producing painful erosions. Nikolsky's sign, in which rubbing of apparently unaffected skin causes a separation of the outer layers, is a helpful diagnostic clue. Pemphigus tends to begin in the mouth, where rapid rupture of the blisters may lead to the impression that the initial lesion is an ulcer rather than a blister. Biopsy with immunofluorescence studies can demonstrate blister formation high in the epithelium that is accompanied by IgG deposition on epithelial cell surfaces. Pemphigus vulgaris is a serious, chronic condition that can be life-threatening as a result of fluid/electrolyte imbalance, secondary infection, or complications of the high-dose corticosteroid therapy that may be necessary to bring the condition under control. Bullous pemphigoid (choice A) causes tense bullae that do not rupture easily. Muscosal involvement is rare and lesions are pruritic. Dermatitis herpetiformis (choice B) causes clusters of intensely pruritic vesicles, papules, and urticarial lesions on the extensor surfaces unlike pemphigus vulgaris which mainly affects the flexural surfaces. Stevens-Johnson syndrome (SJS) (choice D) is a severe, blistering, form of erythema multiforme that characteristically shows at least a few target lesions. A history of drug ingestion or infection (usually a flu-like prodrome) should be present in both SJS and toxic epidermal necrolysis (TEN) with rapid onset of skin lesions. This was not described in the case above, making the diagnosis unlikely. Toxic epidermal necrolysis (TEN) (choice E) can cause widespread flaccid blisters and Nikolsky's sign may be positive, but this disorder tends to develop much more rapidly than pemphigus vulgaris. Clinical Pearls Pemphigus vulgaris is a rare disease, but the most common form of pemphigus in the United States and Europe. Most commonly seen in middle-aged and older patients, it is an autoimmune, bullous disease which causes blisters to form as a result of loss of cohesion between the epidermal cells. Pemphigus vulgaris can affect both cutaneous and mucosal surfaces and usually presents with flaccid bullae that easily rupture leading to painful erosions. Diagnosis requires biopsy from both involved skin and from normal appearing skin surrounding the lesions. Patients may have a positive Nikolsky's sign in which there is epidermal sloughing when traction is placed on normal skin. Treatment is systemic glucocorticoids or other immunosuppressive agents (azathioprine, cyclophosphamide); for refractory disease, treatment is rituximab and IV immunoglobulin.

A 48-year-old man comes to the healthcare provider because of a skin rash his girlfriend noticed on his chest and back. He is not sure how long it has been there. He is in good health and routinely takes vitamin supplements. His job is very demanding and he works out in a gym 5 nights a week to stay fit and alleviate stress. After his girlfriend commented on the rash he did notice that he has lately started feeling slightly itchy after working out, but did not make anything of that. He has no history of allergies to medications and the family history is negative for skin diseases. On physical examination, the patient is a muscular, fit man in no acute distress. On the sides of the neck, anterior chest, shoulders, and upper back, there are many round and oval, discrete and confluent, hypopigmented patches with a barely visible fine scale that is accentuated by rubbing. An image of the lesion is shown. The rest of the physical examination is unremarkable. Which of the following is most likely mechanism of the pigmentary change? A. Contact with new laundry detergent B. Sexually spread infection C. Yeast infection of the skin D. Bacterial infection of the skin E. Self-inflicted physical trauma

The correct answer is C. The patient has tinea versicolor, a common skin infection in young adults who perspire freely. Tinea versicolor is caused by a nondermatophyte, dimorphic fungus that is a normal inhabitant of the skin. As the yeast form Pityrosporum orbiculare, it generally does not cause disease (except for folliculitis in certain cases). However, in some individuals it converts to the hyphal form, Malassezia furfur, and causes characteristic lesions. It favors heat and humidity, and is commonly seen in people who engage in strenuous physical activity. It is characterized by oval scaly macules, papules, and patches most commonly on the chest, shoulders, and back with hypopigmentation on dark skin and slight hyperpigmentation on light skin. A potassium hydroxide preparation from scaling lesions will demonstrate short hyphae and round spores ("spaghetti and meatballs"). Topical selenium sulfide solution (in the form of a shampoo) is the drug of choice. Treatment with oral ketoconazole is also effective. Topical antifungal creams are an alternative but not favored as first line therapy because of the large body surface area that requires application. Hypopigmentation will take some time to resolve even after successful treatment. Contact with a new laundry detergent (choice A) can lead to contact dermatitis in susceptible individuals but this would cause papules and urticarial with pruritus being a common complaint. The poxvirus can be spread sexually (choice B) and causes molluscum contagiosum. The appearance of this skin condition is lesions that are small, domed, umbilicated lesions. Bacterial infection (choice D) can occur on the skin and cause erysipelas and cellulitis. These infections cause redness to the skin with either a distinct border (erysipelas) or an indistinct border (cellulitis). This patient has hypopigmented lesions. Self-inflicted physical trauma (choice E) can result in hypopigmented or hyperpigmented macules and atrophic scars on areas of the body within easy reach of the hands. This disorder, called neurotic excoriations, is most commonly seen in young or middle-aged women and in patients with posttraumatic stress disorder. There are no primary lesions, only monomorphous excoriations and scarring with postinflammatory pigment disorder. Clinical Pearls: Tinea versicolor is caused by the nondermatophyte dimorphic fungus Malassezia furfur (Pityrosporum orbiculare). It causes depigmentation due to inhibition of tyrosinase and toxicity to melanocytes caused by the dicarboxylic acid azelaic acid, which is produced by the hyphal form of the organism. If large amount of skin is involved, systemic treatment with ketoconazole can be more effective than topical therapy.

A 16-year-old girl is brought to the emergency department by ambulance after she was extracted from a burning vehicle that had been in an accident on the local highway. She was the only person in her car, which burst into flames after being rear-ended at high speed. She swayed off the road and collided with the side railing. When the ambulance arrived, she was unconscious in the driver's seat. On arrival at the emergency center, the patient is in moderate respiratory distress. Her pulse is 120/min, blood pressure is 80/40 mm Hg, and respirations are 30/min. After securing the airway and administering oxygen and IV fluid, the healthcare provider evaluates the extent of the sustained burn injury. There is diffuse erythema and edema of her face and most of her scalp hair is scorched, with some blistering of the underlying skin. Both upper extremities circumferentially show diffuse erythema, edema, and areas of extensive blistering. The rest of her body shows no significant burns. Which of the following is the estimated body surface area of the burn? A. 9% B. 18% C. 27% D. 36% E. 45%

The correct answer is C. This patient has sustained first-degree burns of the face and superficial second degree burns of the scalp and both arms, for a total of 27% estimated body surface area involvement based on the "rule of 9s." When evaluating a burn victim, estimation of body surface area of the burn should be performed. This can be done by using burn charts for children of different ages up age 14, or by means of the "rule of 9s" used in adults and children age ≥14. The "rule of 9s" allows a quick estimate for preliminary calculations of the required volume of resuscitation fluid. Head and arms = 9% each Legs = 18% each Trunk = 36% (18% each for front and back) The "rule of palm" may be used for burns that involve <10% body surface area, in which the child's palm equals 1% of the child's body surface area. Clinical Pearls There are several methods to estimate the total body suface area (TBSA) affected with burns. The easiest way to estimate the TBSA is the "rule of 9s," which is used in adults: Each lower extremity represents 18% TBSA Each upper extremity represents 9% TBSA Anterior and posterior trunk each represent 18% TBSA Head represents 9% TBSA The "rule of 9s" is less accurate in children since they have proportionately larger heads compared to their extremities. Smaller burns can be estimated using the "rule of palm," which estimates the entire palm surface of the hand, including fingers, as 1% of TBSA. This technique is accurate in both children and adults.

A 15-year-old boy comes to the healthcare provider for advice about his facial acne. He is very concerned with his appearance and when questioned, he says that he washes his face 2 times a day with a very mild soap. He tries to eat a healthy well-balanced diet but loves chocolate and spicy foods. He plays on the high school junior varsity basketball team. On examination, the patient has mild to moderate acne, mostly consisting of open comedones, some closed comedones, and a few pustules on the forehead and cheeks. The remainder of the examination is unremarkable. Which of the following is the best advice to give this patient? A. Avoidance of chocolate and spicy foods B. Frequent face washing with strong soap C. Oral treatment with isotretinoin D. Topical application of tretinoin E. Treatment with oral antibiotic

The correct answer is D. Acne vulgaris affects the majority of adolescents and is more prevalent in males. Hormonal influences, abnormal keratinization of pilosebaceous units and colonization by bacteria (Propionibacterium acnes) are important pathogenetic elements. Treatment of acne depends on the severity of the condition. Retinoids (e.g., tretinoin), a topical comedolytic agent, is effective for mild to moderate forms of noninflammatory acne, which is characterized by open comedones. Daily application of these compounds will result in improvement within several weeks after starting treatment. Mild skin irritation and scaling may be minimized by starting with low-concentration creams, and then progressively increasing the concentration. Another side effect of retinoids is increased photosensitivity. The patient must be instructed to avoid prolonged exposure to the sun and to use sunscreen. Avoidance of chocolate and spicy foods (choice A) would have no beneficial effects on acne. It is well established that there is no correlation between acne and specific types of foods. Frequent face washing with strong soap (choice B) will probably cause exacerbation of acne. This skin condition is not caused by dirt. Gentle face washing once or twice daily with mild soaps is recommended. Oral treatment with isotretinoin (choice C) is used for severe cases of acne not responding to topical comedolytics and antibiotics. This compound acts by decreasing sebum production. In addition to various types of side effects, isotretinoin is teratogenic. Female patients of childbearing age are required to use effective means of contraception beginning one month before treatment to several months after treatment. Special licensing is needed to prescribe this medication due to restricted distribution of this medication. Treatment with oral antibiotic (choice E) is aimed at decreasing bacterial colonization It is used for patients who fail to respond to topical treatments or have severe forms of inflammatory acne. The antibiotics of choice include tetracycline, doxycycline, and minocycline. Clinical Pearls Acne vulgaris is one of the most common presenting teenage complaints. Acne is a disorder of the pilosebaceous follicles in which there is follicular hyperkeratinization, sebum overproduction, presence of Propionibacterium acnes, and inflammation as part of the pathophysiology. Acne is more common in adolescents but can also occur in adults. It is not physically debilitating, but it can have significant psychological consequences (low self-esteem, anxiety). Therefore, there is a large demand for effective treatment options. Topical retinoids, benzoyl peroxide, azelaic acid, and salicylic acid are all effective against comedonal acne with the topical retinoids being considered as first-line therapy. Mild to moderate inflammatory acne should be treated with topical retinoids, topical antibiotics, and benzoyl peroxide. Addition of topical antibiotics to benzoyl peroxide is superior to either agent alone and use with benzoyl peroxide decreases the incidence of antibiotic resistance. Moderate to severe inflammatory acne should be treated with topical retinoids, topical benzoyl peroxide, and an oral antibiotic. Oral antibiotics should be given for 3-5 months. Topical benzoyl peroxide should be used concomitantly with oral antibiotics to decrease the incidence of antibiotic resistance. Severe nodular acne is treated with oral isotretinoin, but this medication falls under a special distribution license.

A 35-year-old woman consults a healthcare provider about a lesion on her finger that developed after a minor injury. The lesion is a scarlet, apparently vascular, raised nodule that bleeds easily and does not blanch on pressure. No inflammatory changes are seen in the surrounding skin and the woman reports that the lesion is not uncomfortable, although its cosmetic appearance does bother her. Which of the following is the most likely diagnosis? A. Dermatofibroma B. Keloid C. Keratoacanthoma D. Pyogenic granuloma E. Wart

The correct answer is D. All of the skin lesions listed are common, minor skin problems that you will need to distinguish clinically. This woman has a pyogenic granuloma. This lesion most commonly develops at a site of previous injury, and is best considered a variation of granulation tissue formation. The name is a misnomer, since the lesion is neither related to bacterial infection nor a true granuloma. Microscopically, proliferating blood vessels in an immature fibrous stroma are seen. The lesions can occasionally be confused with dysplastic nevi or malignant melanoma, but are completely benign. Treatment is surgical excision, curettage, or electrodessication. A dermatofibroma (choice A) is a red to brown, firm, small papule or nodule, often found on the legs. A keloid (choice B) is a shiny, hard, smooth, often dome-shaped overgrowth of a scar. A keratoacanthoma (choice C) is a round, firm, flesh-colored nodule with a central crater. A wart (choice E) is a rough-surfaced, sharply demarcated, round, firm, nodule whose color may be gray, yellow, brown, or gray-black. Clinical Pearls Pyogenic granulomas are benign capillary vascular proliferations that occur at sites of minor trauma. They are characterized clinically by a rapidly enlarging, red, vascular papule with a surrounding thin scale. Pyogenic granulomas are benign skin lesions with no malignant potential.

Bowen disease is a superficial skin cancer of which type? A. Actinic keratosis B. Melanoma C. Basal cell D. Squamous cell

The correct answer is D. Bowen disease is a superficial erythematous patch or plaque, often with a scale, that has involvement limited to the epidermis and is considered to be squamous cell carcinoma in situ. Bowen disease is a superficial erythematous patch or plaque, often with a scale, that has involvement limited to the epidermis and is considered to be squamous cell carcinoma in situ, not actinic keratosis (choice A). Bowen disease is a superficial erythematous patch or plaque, often with a scale, that has involvement limited to the epidermis and is considered to be squamous cell carcinoma in situ, not melanoma (choice B). Bowen disease is a superficial erythematous patch or plaque, often with a scale, that has involvement limited to the epidermis and is considered to be squamous, not basal (choice C), cell carcinoma in situ.

A patient presents to the office with concerns about a skin lesion on her face. The lesion is an ulcerated nodule. Which of the following is the most likely diagnosis? A. Normal finding B. Melanoma C. Basal cell carcinoma D. Squamous cell carcinoma

The correct answer is D. Squamous cell carcinoma may appear as a hyperkeratotic papule, erosion, or nodule (which may be ulcerated). Normal findings (choice A), melanoma (choice B), and basal cell carcinoma (choice C) do not appear as ulcerated nodules.

Which of the following types of skin cancer is most likely to occur after a previous burn to the skin or along a scar ("scar carcinoma")? A. Actinic keratosis B. Melanoma C. Basal cell carcinoma D. Squamous cell carcinoma

The correct answer is D. Squamous cell carcinoma predisposing factors include fair complexion, chronic UV exposure, and previous burn or other scar. Actinic keratosis (choice A), melanoma (choice B), and basal cell carcinoma (choice C) are not most likely to occur after a previous burn to the skin or along a scar.

Which of the following types of malignant melanoma has the worst prognosis when it is identified? A. Superficial spreading B. Lentigo maligna melanoma C. Acral lentiginous D. Nodular

The correct answer is D. The nodular type of malignant melanoma has the worst prognosis because it has invasive growth from its onset. Superficial spreading (choice A), lentigo maligna melanoma (choice B), and acral lentiginous (choice C) subtypes of malignant melanoma do not carry the worst prognosis,

A 35-year-old woman consults a healthcare provider about a changing mole on her neck. The lesion is a mostly flat, 2-cm diameter patch with varying colors from red to brown to black. In several areas, small nodules are seen within the patch. Much of the peripheral circumference of the lesion has a poorly defined, "feathery" edge. Biopsy demonstrates a malignant tumor of melanocytes. The lesion is resected with a wide resection margin and then examined pathologically. Which of the following is the most significant prognostic indicator in this lesion? A. Area of involved skin B. Degree of atypia of the tumor cells C. Degree of color variation D. Depth of invasion E. Percentage of circumference involved with "feathering"

The correct answer is D. The patient has a malignant melanoma. Melanomas that have not yet metastasized can be simply treated with wide-surgical excision, while melanomas that have metastasized can be fatal due to lack of response to treatment. The single most important prognostic factor for melanoma is the depth of the invasion into the dermis. This is because of a feature of the morphology of the skin: the very most superficial layer of the dermis is relatively free of lymphatics, while a very rich lymphatic bed is found slightly deeper. Melanomas greater than 1 mm in depth are very likely to have penetrated this lymphatic bed and potentially metastasized. Growth of these lesions is not just superficial but also growing deeper which is not apparent to the eye. Area of involved skin (choice A), degree of atypia of the tumor cells (choice B), degree of color variation (choice C), and percentage of circumference involved with "feathering" (choice E) are only indirectly important as they may contribute to the local aggressiveness of the tumor and thus increase the chance of deeper tumor penetration. Clinical Pearls Malignant melanoma is the most dangerous form of skin cancer and the sixth most common cancer in North America. The incidence of melanoma increases with age and sun exposure. Prognosis for patient is dependent on thickness of the lesion, ulceration, mitotic rate, and presence and extent of metastatic disease. It should be noted that the depth of the lesion is the single most important prognostic factor with patients with localized melanoma.

A 42-year-old man consults a healthcare provider because he has a "lump" on his forearm. Examination of the arm demonstrates a soft 3-cm diameter subcutaneous lesion protruding above the forearm surface. The lesion is covered with apparently normal skin and is soft and freely movable. The lesion has been slowly growing over the past 2 years, and the patient has experienced no discomfort. He has consulted a healthcare provider at this time because his wife keeps pestering him to get something done about it. Which of the following is the most likely diagnosis? A. Capillary hemangioma B. Dermatofibroma C. Intradermal nevus D. Lipoma E. Seborrheic keratosis

The correct answer is D. This is probably a lipoma, which is a benign mass lesion composed of mature adipose tissue bound by a limiting membrane. (Another possibility is an epidermoid cyst, which can be indistinguishable clinically from lipoma.) Lipomas are very common, and patients may have more than one lipoma. Common sites include the trunk, nape of the neck, and forearms. The lesions are only rarely malignant, although a rapidly growing lesion should be biopsied to make sure of the diagnosis. They are usually asymptomatic; a small percentage are painful. They can be treated with surgical excision or liposuction. Since these lesions are benign, lipomas may be removed for cosmetic reasons or due to functional impairment due to the size of these benign growths. Capillary hemangioma (choice A), also known as strawberry mark, is a bright red, vascular lesion that usually develops shortly after birth and then often involutes by late childhood. Dermatofibroma (choice B) causes a firm, red to brown, small papule or nodule that is most frequently found on the legs. Intradermal nevus (choice C) causes a flesh colored to black, elevated, lesion that is usually 3 to 6 mm in size. Seborrheic keratosis (choice E) causes a pigmented, superficial, usually warty, epithelial lesion. Clinical Pearls Subcutaneous lipomas are common benign neoplasms of the skin and subcutaneous tissues. These lesions are usually not symptomatic, but are concerning to patients who either think they be cancerous or because of cosmetic concerns. Lipomas are collections of mature adipose cells that are encapsulated by a fibrous wall and can occur anywhere on the body where adipose cells are present. Lesions can present as a soft small "ball" under the skin. Lesions can be solitary or numerous and some patients have familial multiple lipomatosis, a genetic condition that predisposes patients to developing multiple lipomas. It is rare that lipomas can undergo a malignant transformation into a liposarcoma. Biopsy or removal is indicated if lesions are painful, if they are rapidly enlarging, or if they are firm. Treatment for lipomas is surgical excision of the adipose cells as well as the fibrous capsule. Recurrence of a lipoma is not common, but new ones can form. Patients should be warned about scarring, seroma, and possible hematoma formation after excision.

A 73-year-old woman comes to the office accompanied by her daughter for a regular semiannual exam. She has diabetes and hypertension and regularly takes metformin (Glucophage), pentoxifylline (Trental), and nifedipine (Procardia, Adalat). She is concerned about a very dark skin lesion that has been enlarging on her right temple in the previous 6 months. Her daughter insists it has to be biopsied because it may be a skin cancer. Her mother had extensive sun exposure in her youth and has already had a basal cell carcinoma removed from her back 2 years earlier. On examination, the patient is in no acute distress. Vital signs are normal and fasting glucose is within normal limits. On the right temple there is a 2 x 3-cm, dark brown, waxy, keratotic plaque with a verrucous surface punctuated by multiple pseudocysts. It appears to be pasted on her face, and gentle scraping of the surface induces small keratotic debris to dislodge. Which of the following is the most likely diagnosis for this skin lesion? A. Actinic keratosis B. Basal cell carcinoma C. Melanoma D. Seborrheic keratosis E. Solar lentigo

The correct answer is D. This patient has developed a seborrheic keratosis on her right temple. Seborrheic keratoses are oval, slightly raised, tan- to black-colored, sharply demarcated papules or plaques that rarely grow >3 cm in diameter. Most commonly, they are located on the back, chest, face, and scalp, but may also occur on the extremities. A superficial shave biopsy or curettage will provide material for analysis without leaving a considerable scar. Typical seborrheic keratoses may be treated with methods of physical destruction, such as with liquid nitrogen or light electrodesiccation after the bulk of the lesion has been curetted. However, they will commonly recur. An actinic keratosis (choice A) is an ill-demarcated erythematous macule or papule with a sharply rough keratotic surface and adherent scale. Actinic keratoses are usually present in numbers and appear as a consequence of significant cumulative actinic damage to the skin. Scraping an actinic keratosis will induce bleeding. Histopathologic analysis of a shave biopsy sample will provide the diagnosis in clinically atypical lesions. Basal cell carcinoma (choice B) typically presents as a well demarcated pink or pearly-white papule that has a central indention with keratotic debris or a crust. The surface is traversed with multiple telangiectases. Removal of the central crust will reveal a superficial ulceration. Basal cell carcinomas are prone to bleeding, and the patient will usually complain that even minor trauma induces bleeding of the lesion. Melanoma (choice C) presents as a smooth-surfaced, shiny, dark brown to black papule, nodule, or plaque with a slightly infiltrating pattern. The skin markings on the surface are effaced and there is often variegation in color. Irregular borders of the lesion are often scalloped or spiked, and the lesion is asymmetric when an imaginary line is drawn down the middle. Appearance of areas of white or gray within the lesion signifies regression, which is an ominous prognostic sign. Sometimes, an erythematous halo may be present as the result of an inflammatory response. Long-standing melanoma may ulcerate. Solar lentigo (choice E) is frequently present on sun-damaged skin and is characterized by multiple, 1- to 2-mm, well demarcated, tan to brown, uniformly colored macules. Infrequently, a lesion of solar lentigo may grow to be several millimeters or even centimeters in size, but this is atypical and such lesions should be biopsied to exclude melanoma in situ. A history of frequent sun exposure, especially extensive sunburns, is usually elicited. Sites of predilection are the face, neck, anterior chest, upper back, shoulders, and arms. Clinical Pearls Seborrheic keratosis is a benign epidermal tumor common in those over age 50. It can range from a single lesion to hundreds of lesions. The lesions, caused by a hyperproliferation of immature keratinocytes, can be hereditary (autosomal dominant) or can follow the healing phase of an inflammatory skin disease. There is no effective way to prevent the formation of lesions. GI and lungs cancers can be associated with the sudden appearance of multiple seborrheic keratoses, especially if there are concomitant skin tags or acanthosis nigricans. Diagnosis is clinical but atypical lesions may need biopsy. Classic lesions appear as "stuck on" well-circumscribed, scaly, hyperpigmented, warty lesions. They occur most commonly on the trunk, face, or extremities. Treatment is not necessary unless they are symptomatic or for a cosmetic issue in which case they can be excised.

A 68-year-old man is hospitalized because of a 24-hour history of fever and chills and a spreading infection of the lower leg of 3 days' duration. It started as tender swelling and redness on the right foot that subsequently spread to involve the lower half of the shin. The patient denies injury or any unusual activity in the prior week. He has a history of heart disease and hypertension treated with digoxin, losartan (Cozaar), and hydrochlorothiazide. He is plethoric. His blood pressure is 150/90 mmHg, pulse is 95/min and respirations are 28/min. He is 5 feet 10 inches tall and weighs 230 pounds. Examination of extremities reveals bilateral 2+ nonpitting edema of the lower legs and well-demarcated, intense, shiny erythema and edema of the distal anteromedial right shin and foot with a distinct, slightly raised proximal border. The affected area is very warm to touch. Several tense bullae filled with clear fluid are noted on the dorsum of the foot. The toes are edematous and erythematous, with extensive macerated scale in the fourth and fifth interdigital spaces and deep fissures at the base of the affected toe webs. IV antibiotics are initiated and the infection successfully subsides over a 10-day period. Which of the following is the most important step in preventing future recurrences of this disease? A. Compression therapy B. Increasing the dose of the diuretics C. Long-term, low-dose antibiotics D. Topical antifungals and meticulous foot hygiene E. Weight reduction

The correct answer is D. This patient has erysipelas, a superficial cellulitis of the skin with marked lymphatic involvement, caused by Group A beta-hemolytic streptococci. Erysipelas most commonly occurs in infants, very young children, and the elderly. In most cases, the organism gains access by direct inoculation through a break in the skin, but hematogenous infection may occur infrequently. It begins with an area of redness and enlarges to form the typical tense, painful, bright red, shiny, brawny infiltrated plaque with a well-demarcated, distinct, and slightly raised border (as opposed to deeper cellulitis, which has no distinct border and is flush with surrounding skin). Penicillin is the treatment of choice. In people allergic to penicillin, erythromycin, azithromycin, or clindamycin is recommended. In patients with no obvious portal of entry, a hidden site of inoculation must be sought. Frequently, a long-standing fungal infection of the interdigital spaces will have fissures at the base of the toe webs that are easily overlooked. Unless this simmering dermatophytosis is treated, the infection is bound to recur. Topical antifungals are applied for up to 2 weeks beyond clinical cure, and meticulous foot hygiene should be exercised to prevent recurrence of the fungal infection. The interdigital spaces must be clean and dry. With edema of the toes, drying the interdigital spaces with a hair dryer and regularly applying foot powder are very helpful. Compression therapy (choice A) is imperative in treating chronic vascular insufficiency of the lower extremities with or without ulceration. Compressive stockings or elastic wraps are applied in the morning and removed at nighttime. Although they may help with this patient's leg edema and indirectly decrease the occlusive environment created by the edematous toes, they would not aid in the resolution of the fungal infection without appropriate topical antifungal therapy. Increasing the dose of the diuretics (choice B) may alleviate the leg edema and, similar to the compression therapy, decreases the environmental factors contributing to this patient's fungal infection, but it would not prevent future recurrences of cellulitis if not accompanied by antifungal treatment. Long-term, low-dose antibiotics (choice C) are seldom used except for a few chronic inflammatory disorders, such as acne or sarcoidosis, where their anti-inflammatory (as opposed to antibacterial) properties are used. Tetracycline antibiotics are most commonly prescribed this way. Initially, a full antibacterial dose is administered, and with improvement, tapered to the lowest dose that controls the disease. They have no place in the prophylaxis of erysipelas. Weight reduction (choice E) would benefit this patient's heart disease, hypertension, and overall health, but it would not influence recurrence of erysipelas if the interdigital skin barrier is not normalized. Clinical Pearls: Cellulitis and erysipelas are both infections of the skin that are characterized by erythema, swelling, and warmth without any underlying suppurative foci. The main difference between these 2 infections is that erysipelas tends to be raised and has a clear demarcation between it and the surrounding skin. Both are usually caused by some disruption to the skin barrier. This can occur with trauma, inflammation, pre-existing skin infection, or edema. Erysipelas affecting the face is most commonly caused by staph aureus while erysipelas of the lower extremities is most commonly caused by strep. Erysipelas tends to be more acute in onset when compared to cellulitis, and patients will have more sudden systemic manifestations such as fever and chills. Treatment for both cellulitis and erysipelas includes rest, elevation, and treatment of the underlying cause. Hydrating the skin is critical to prevent cracks or fissures, but taking care to not macerate the skin is also important. Treatment of tinea pedis, lymphedema, and chronic venous insufficiency is also important in order to prevent secondary bacterial infection.

A 35-year-old man presents to the healthcare provider with intensely pruritic skin lesions on his axillary folds, nipples, waistband, wrists, interdigital spaces of the hand, and genitalia. Linear burrows are apparent in affected areas. Which of the following is the most likely diagnosis? A. Dermatitis herpetiformis B. Lichen planus C. Photosensitivity D. Scabies E. Seborrhea

The correct answer is D. This pattern of lesions is seen in scabies, a contagious skin disease characterized by intense itching. Scabies is caused by infestation with small, burrowing mites (Sarcoptes scabiei). Dermatitis herpetiformis (choice A) typically affects the scalp, shoulders, anterior surface of the knees, elbows, and the small of the back. Lichen planus (choice B) affects the mouth, anterior forearms, genitalia, the small of the back, and the posterior aspect of the leg below the knee. Photosensitivity (choice C) produces skin changes in the face and anterior neck except below the chin, the back of the neck, and the dorsum of the hands. Seborrheic rashes (choice E) affect the scalp, posterior surface of the head, neck, and shoulders, the perianal region, chin, groin, umbilicus, and sternum. Clinical Pearls Scabies is caused by the mite Sarcoptes scabiei. It is usually transmitted by direct contact with an infected person but can also be transmitted on fomites (clothing, linen, etc.). Patients typically present with intense itching that tends to be worse at night. Diagnosis is usually clinical in a patient with a pruritic rash with the characteristic distribution and cohabitants with similar symptoms. Treatment is topical permethrin 5% cream. To prevent transmission, close inhabitants should also be treated and all clothing/linens should be bagged for several days and then washed in hot water and dried with high heat.

A 30-year-old football player lacerates his face in a fall during practice. He is taken to a healthcare provider, who finds a clean wound with well-approximated edges. There is little surrounding tissue damage. Which of the following stages of wound healing will occur first? A. A thin, continuous epithelial cover will appear B. Fibroblasts will lay down collagen fibers C. Granulation tissue will fill the wound D. Macrophages will appear E. Neutrophils will line the wound edge

The correct answer is E. Neutrophils line the wound edge within 24 hours of injury. A thin, continuous epithelial cover appears (choice A) 24 to 48 hours after injury. Soon after, macrophages replace the neutrophils (choice D), while granulation tissue fills the wound (choice C). By day 5, fibroblasts lay down collagen fibers (choice B) along fibrin and fibronectin matrices formed across the site of incision. Full maturation of the scar requires up to 1 year. Clinical Pearls Wound healing occurs in stages or phases: Hemostasis: Immediately after an injury platelets start to aggregate and trigger the clotting cascade. The resulting fibrin matrix stabilizes the wound and provides the scaffolding for the wound healing process. Inflammation: The inflammatory phase usually takes about 3 days and is characterized by mononuclear leukocytes accumulating and transforming into macrophages. This phase may be prolonged with infection or chronic wounds. Epithelialization: During this phase, basal cell proliferation and epithelial migration lead to a superficial layer of epithelium which creates a barrier against bacteria and other foreign material. Migration ceases after about 48 hours from when the injury occurred. Fibroplasia: Fibroblasts proliferate, accumulate a ground substance, and collagen is produced which stimulates angiogenesis leading to the formation of granulation tissue (collagen plus growth of capillaries). This phase starts within 24 hours of the wound and predominates by day 10. Maturation: Maturation of a wound can take several months and represents collagen cross-linking, collagen remodeling, wound contraction, and repigmentation. Tissue strength is regained about 6 weeks after the incident.

A 30-year-old man presents to the healthcare provider with silvery, scaling plaques on his elbows and knees. His mother was afflicted with the same condition in the past. Which of the following is the most likely diagnosis? A. Acne rosacea B. Acne vulgaris C. Pemphigus vulgaris D. Pityriasis rosea E. Psoriasis vulgaris

The correct answer is E. Psoriasis vulgaris usually appears on the nails, knees, elbows, and scalp. It does not generally affect the mucous membranes. Lesions are well-demarcated, coral-colored plaques with a white or silvery scale. Histologically, epidermal hyperplasia causing thickening and lengthening of the rete ridges is apparent, as is thinning of the epidermis over the dermal papillae. There is a recognized genetic component to this condition. Peak incidence is at age 30. Acne rosacea (choice A) affects the central face. Erythema, telangiectasias, acneform lesions (papules, cysts, pustules), and rhinophyma (telangiectasias and hyperplasia of nasal soft tissue) are found in various combinations. It is common from ages 30 to 50. Women are affected 3 times more frequently than men, but the syndrome is more severe in the latter. Acne vulgaris (choice B) causes comedones, papules, and cysts. It may be related to hormones, drugs, diet, irritants, and genetic factors. Allergy to Propionibacterium acnes has been found to contribute to this condition. Pemphigus vulgaris (choice C) starts with small vesicles, usually on the oral or nasal mucosa, then spreads to other parts of the body. Bullae are delicate and flaccid. The condition is due to autoantibodies to intercellular junctions between keratinocytes. Nikolsky's sign (production of blistering by light stroking or rubbing of the skin) is positive. Pemphigus is most common age 40-60. Pityriasis rosea (choice D) presents first with a red, scaling herald patch approximately 4 cm in diameter, followed within days by eruption in a short-sleeve turtleneck distribution. The classic clue to the diagnosis is the appearance of crops of small, pink, oval patches in a Christmas tree configuration on the flexural lines. Clinical Pearls Psoriasis is a chronic, immune-mediated skin disease that causes erythematous papules or plaques with a silver scale. Peak incidence is bimodal at age 20-30 and also age 50-60. It affects both sexes equally. Some studies indicate there may be a correlation between smoking, obesity, and alcohol consumption. Mild to moderate disease should be treated with topical steroids and emollients, while more severe disease should be treated with phototherapy if possible. Patients with a history of skin cancer or psoriatic arthritis should be treated with a systemic agent such as a retinoid, methotrexate, cyclosporine, or DMARD. Conditions usually improve in a couple of weeks but total resolution of all lesions is unlikely to occur.

A 60-year-old man comes to the office because of a reddish, scaling lesion on his lower lip that has been growing for the past year. It is painless and he is here now because his wife made him come. He has not seen a healthcare provider for the past 4 years. He works as a farmer and is exposed to the sun. On physical examination, his lower lip is fixed to the lower aspect of his mandible. Which of the following is the correct diagnosis? A. Basal cell carcinoma B. Erythema nodosum C. Leukoplakia D. Melanoma E. Squamous cell carcinoma

The correct answer is E. Squamous cell carcinoma is the second most common form of skin cancer after basal cell carcinoma and is the most common type of cancer on sun-exposed surfaces in older individuals. The location on the lower lip is especially suggestive of squamous cell carcinoma. Basal cell carcinoma (choice A) is the most common form of skin cancer; risk factors include sun exposure and ultraviolet radiation. Features include an ulcerated center with pearly, heaped up edges. Erythema nodosum (choice B) is a vascular disorder whose skin manifestations include erythematous and nodular lesions, especially on the anterior aspect of the tibia. Leukoplakia (choice C) is an Epstein-Barr virus-related lesion that would be found on the lateral aspects of the tongue and is white in appearance. Melanoma (choice D) has the maximum metastatic potential of any skin cancer, and may be nodular or radial. It may have irregular borders, may be of variegated coloration, and may be on any aspect of the body. Clinical Pearls Squamous cell carcinoma (SCC) is the second most common type of skin cancer in the United States and accounts for about 20% of nonmelanoma skin cancers. SCC has a predilection for sun exposed areas of skin, especially in fair-skinned patients, but any skin surface can be involved, especially in darker-skinned patients. SCC typically presents as an erythematous papule, plaque, or nodule which may or may not have hyperkeratosis, ulceration, or hyperpigmentation. SCC typically presents as an erythematous papule, plaque, or nodule which may or may not have hyperkeratosis, ulceration, or hyperpigmentation.

A man who weighs 65 kg sustains second and third degree burns over both of his lower extremities when his pants catch on fire. When examined shortly thereafter, it is ascertained that virtually all of the skin from both groins to the tip of the toes, front and back, has been burned. According to the Parkland formula, which of the following is the approximate total amount of IV fluid that he can be expected to require during the first 24 hours post-burn? A. 3460 mL B. 4960 mL C. 6760 mL D. 8160 mL E. 9360 mL

The correct answer is E. The Parkland formula calls for 4 mL of Ringer's lactate per kilogram of body weight, times the percentage of the body surface that has been burned. In the "rule of 9s," each lower extremity represents 18% of the body surface. Thus, this patient has sustained a 36% body burn: 4 × 65 × 36 = 9360 mL. None of the other options provide enough fluid, although in the real world the formula calculations are used to help determine a "ballpark figure" and a rate of initial infusion. Once fluid is going in, fine-tune the infusion rate on the basis of hourly urinary output and central venous pressure. Clinical Pearls Parkland Formula = 4 mL of LR (% TBSA burned) (wt in kg), with half given over the first 8 hours and the remainder over the following 16 hours. The time starts from when the burn was sustained and not from when the fluid resuscitation was initiated. In patients who have experienced burns on both lower extremities, the TBSA burned is 36% (18% involvement with each lower extremity). Total crystalloid for first 24 hours = 4 mL (36%)(65 kg) = 9,360 mL. This patient would then receive 585 mL/hr for the first 8 hours (4,680 total mL), and 292.5 mL/hr for the next 16 hours (4680 total mL).

A 65-year-old, fair-skinned man develops scaly erythematous papules that have a sandpaper texture on his forehead. A biopsy shows nuclear crowding, hyperchromasia, and pleomorphism of the basal keratinocytes, with alteration of the normal epidermal maturation. Hyperkeratosis is also present. Which of the following neoplasms may arise from this lesion? A. Basal cell carcinoma B. Melanoma C. Merkel cell carcinoma D. Mycosis fungoides E. Squamous cell carcinoma

The correct answer is E. The lesion described is actinic keratosis, which is frequent on sun-exposed areas of people of fair complexion. It is related to progressive dysplastic changes triggered by the carcinogenic action of ultraviolet light. Actinic keratosis is a premalignant change, but only a small number of untreated lesions will eventually progress to invasive squamous cell carcinoma. Basal cell carcinoma (choice A) is a common skin neoplasm with minimal metastatic potential. It is also most frequent on sun-exposed areas, but actinic keratosis is not a precursor. As with squamous cell and basal cell carcinomas, sunlight is an important predisposing condition for the development of skin melanomas (choice B), but actinic keratosis is not. Besides sunlight, preexisting nevi, genetic factors, and possibly chemical carcinogens play a pathogenetic role. Merkel cell carcinoma (choice C) is a rare neoplasm that is morphologically identical to small cell carcinoma of the lung. It originates from the epidermal Merkel cells. No premalignant precursor is known for this cancer. Mycosis fungoides (choice D) is a malignant T-cell lymphoma that arises from the skin manifesting with scaly patches, plaques, and nodules. Malignant lymphocytes accumulate underneath the epidermis and infiltrate the overlying epidermis. Clinical Pearls Actinic keratoses (AKs) are common skin lesions that form from the hyperproliferation of atypical epidermal keratinocytes. Actinic keratoses (AKs) are common skin lesions that form from the hyperproliferation of atypical epidermal keratinocytes. The lesions typically present as a scaly, erythematous macule or papule in areas of sun exposure (scalp, face, lateral neck, dorsum of hands and forearms). AKs can progress to squamous cell carcinoma (SCC). Treatment includes topical medications, photodynamic therapies, destructive therapies, and surgical excision.

A blond, blue-eyed, 69-year-old sailor has a non-healing, indolent, 1.5-cm ulcer on the lower lip, arising from the vermilion border. The ulcer has been present and growing for the past 8 months. He is a pipe smoker, but has no history of alcohol or drug abuse. Physical examination shows sun damaged facial skin, but no other ulcers. There are no enlarged lymph nodes in his neck. Which of the following is the most likely diagnosis? A. Adenocarcinoma B. Basal cell carcinoma C. Benign ulceration due to chronic trauma D. Invasive malignant melanoma E. Squamous cell carcinoma

The correct answer is E. The location and history are classic for squamous cell carcinoma of the lower lip. The absence of metastatic nodes does not invalidate the diagnosis, as most cancers in this location do not metastasize until quite late. Adenocarcinoma (choice A) would be very rare in the lower lip. Basal cell carcinoma (choice B) favors the upper part of the face, above a horizontal line drawn across the mouth. Benign ulceration (choice C) is always a possibility, but it would be a terrible mistake to make such an assumption. Cancer should be a consideration with any patient who presents with a skin complaint. Melanoma (choice D) is again very rare in this location. A history of a pigmented lesion that underwent changes in color, appearance, or diameter would have been suggestive of melanoma as the cause. Clinical Pearls Squamous cell carcinoma (SCC) is the second most common type of skin cancer in the United States and accounts for about 20% of nonmelanoma skin cancers. SCC has a predilection for sun exposed areas of skin, especially in fair-skinned patients, but any skin surface can be involved, especially in darker-skinned patients. SCC typically presents as an erythematous papule, plaque, or nodule that may or may not have hyperkeratosis, ulceration, or hyperpigmentation. SCC of the lip is most common on the lower lip and may present as a nodule, ulcer, or indurated white plaque. This location is most concerning for spread of this cancer.

A 21-year-old man with no significant past medical history comes to the healthcare provider complaining of a 4-day history of fevers, chills, severe headache, and diffuse aches and pains. He denies cough. After approximately 48 hours with these symptoms, he developed a rash on the palms of his hands and the soles of his feet and on his wrists and ankles. Over the next 48 hours, the rash began to spread to his arms, legs, and trunk. The rash also changed from maculopapular in nature to petechial. He recently returned from a camping trip to several Western states where he believes he was bitten by 2 ticks. Physical examination reveals a moderately "toxic" appearing man. His temperature is 39.0° C (102.3° F), blood pressure is 110/60 mm Hg, pulse is 113/min, and respirations are 15/min. He has a diffuse rash on his body, including his hands and feet as shown. He does not have photophobia or nuchal rigidity. His heart is tachycardic without murmurs. The remainder of his physical examination is unremarkable. Laboratory results reveal a normal white count and liver function studies. Which of the following is the most likely pathogen? A. Bartonella henselae B. Borrelia burgdorferi C. Coxiella burnetii D. Ehrlichia species E. Rickettsia rickettsii

The correct answer is E. These are classic findings of Rocky Mountain spotted fever (RMSF), which is caused by the small, gram-negative, intracellular organism called Rickettsia rickettsii. RMSF is a tick-borne disease carried by the wood tick and the dog tick. It is most easily recognized by its characteristic rash that starts on the hands and feet and spreads to the trunk and face later. It is also unique in that it causes a vasculitis. Common associated symptoms include fevers, headaches, and general malaise. Patients often appear sicker than their lab study results suggest. Doxycycline is the treatment of choice. Bartonella henselae (choice A) is the organism responsible for bartonellosis or cat scratch disease. Cat scratch disease is spread by way of cat scratch or bites. Typically, patients develop a papule or an ulcer at the site of the inoculation. Approximately 1 to 3 weeks later, patients develop fever, headache, and malaise. Regional lymphadenopathy is common. Lyme disease, another tick-borne disease, is caused by Borrelia burgdorferi (choice B). It is carried by the deer tick. The rash seen with Lyme is known as erythema migrans; it is circular with an area of central clearing. Coxiella burnetii (choice C) is a rickettsial infection that is transmitted by exposure to sheep, cattle, or goats. It causes Q fever. After exposure, an incubation period of 1 to 2 weeks is common. After incubation, patients present with fevers, muscle pain, and cough. Patchy infiltrates are often seen on chest x-ray. Ehrlichia species (choice D) are the tick-borne pathogens that cause ehrlichiosis. This is a disease characterized by fever and headache. A rash is uncommon. It is mostly seen in the Southwestern states. The infection is self-limited and usually benign. Elevated liver function tests and low white blood cell count are commonly seen. Doxycycline can be used to decrease the duration of symptoms. Clinical Pearls: RMSF is a tick-borne illness, most commonly seen in the spring and early summer, with potential fatal consequences. About 33% of patients do not recall a tick contact. R. rickettsii is the causative organism of RMSF. R. rickettsii is an obligate intracellular parasite which is a weakly gram-negative nonmobile coccobacillus. Initially patients may present with a flu-like illness with fever, headaches, fatigue, myalgias, arthralgias, and nausea. Children may present with abdominal pain. The rash of RMSF does not usually present until about day 4 of illness. Diagnosis is clinical as there is no reliable diagnostic test in the early phase of the illness when therapy should be started. Treatment of choice for RMSF is doxycycline.

A 68-year-old man comes to the healthcare provider because of a lesion on the side of his forehead. He has had the lesion for "many years". He is in the office today and says, "My daughter's wedding is in 2 months, and my wife wants me to have this ugly thing removed." He plays golf on a daily basis since retiring 3 years ago. On physical examination, the lesion is a 1.5 cm brown plaque with an adherent greasy scale that appears to be placed onto his face. Which of the following is the most likely diagnosis? A. Actinic keratosis B. Basal cell carcinoma C. Dermatofibroma D. Melanoma E. Seborrheic keratosis

The correct answer is E. This patient has a seborrheic keratosis. This lesion is characterized by light brown to black papules or plaques with an adherent waxy, greasy scale. The "stuck-on" appearance is very characteristic. It is most often found on the face and trunk. Actinic keratosis (choice A) is characterized by flesh-colored or red-to-brown macules or papules with a rough scale. It is most common on sun-exposed skin. Basal cell carcinoma (choice B) is characterized by a papule with a central ulceration and a pearly, raised border. It is most common on sun-damaged skin. Dermatofibroma (choice C) is characterized by a firm, red-to-brown nodule with a dimpling of the skin. It can occur anywhere. Melanoma (choice D) is characterized by an asymmetric pigmented lesion with an irregular border. Any change in an existing skin lesion should be evaluated for melanoma. Clinical Pearls Seborrheic keratosis is a common benign epidermal tumor in patients over the age of 50, but it can occur in young patients. The number of lesions can range from single lesions to hundreds of lesions. The lesions are caused by a hyperproliferation of immature keratinocytes. The lesions can be hereditary (autosomal dominant) or follow the healing phase of an inflammatory skin disease. There is no effective way to prevent the formation of lesions. GI and lungs cancers can be associated with the sudden appearance of multiple seborrheic keratoses, especially if there are concomitant skin tags or acanthosis nigricans. Diagnosis is clinical but atypical lesions may need biopsy. Classic lesions appear as "stuck on" well-circumscribed, scaly, hyperpigmented, warty lesions. They occur most commonly on the trunk, face, or extremities. Treatment is not necessary unless they are symptomatic or a cosmetic issue in which case they can be excised.

A 35-year-old woman comes to the healthcare provider complaining of itchy and painful skin lesions on her hands. The lesions developed after she changed her place of work; she used to work in a small hair salon but recently got a new position at a larger, more popular salon. She is thrilled that such a professional opportunity has come her way, and the potential of not being able to stay because of her skin condition is devastating. When the healthcare provider inquires what substances she typically handles, she replies that she does a lot of hair dyeing as well as perming. Some of the time she uses gloves, but she often forgets. On physical examination several well defined, erythematous, and edematous patches are seen on the fingers and dorsal hands, which are moist and covered by abundant scale-crust. The tips of the fingers are tender, edematous, and fissured. Her fingernails have many horizontal ridges and the cuticle is gone. The perionychium is erythematous and edematous. Her eyelids are also swollen, erythematous, and scaly. Which of the following substances is most likely causing this patient's skin condition? A. Ammonium thioglycolate B. Chromates C. Nickel D. Mercaptobenzothiazole E. Paraphenylenediamine

The correct answer is E. This woman has developed allergic contact dermatitis, most likely to paraphenylenediamine in hair dyes. Permanent types of hair dyes incorporate paraphenylenediamine, a potent sensitizer that may cross-react with many chemicals. Exposure to paraphenylenediamine is the third most common cause of contact dermatitis in cosmetics, after fragrances and preservatives. Ammonium thioglycolate (choice A) is used for alkaline permanent-wave solutions and rarely, if ever, sensitizes. Usually it causes only hair breakage and irritation. The hot type (or acid) perm, first used in the United States in 1973, is far more sensitizing. It contains glyceryl thioglycolate and is no longer used. Cosmetologists and hairdressers are at risk of developing hand dermatitis. The glyceryl thioglycolate persists in the hair for 3 months after application and may cause a long-lasting allergic contact dermatitis. It readily penetrates rubber and vinyl gloves. Chromates (choice B) are strongly corrosive and irritating to the skin. They may act as primary irritants or as sensitizers producing allergic contact dermatitis. They are encountered among tanners, painters, dyers, photographers, polishers, welders, aircraft workers, and diesel engine workers. Traces of chromates in shoes or gloves may cause eczema of the feet and hands. The skin changes range from a mild follicular dermatitis to widespread nodular and crusted eruptions, all being worse on exposed parts. Nickel (choice C) dermatitis is a common occurrence, especially among young women. Nickel produces more cases of allergic contact dermatitis than all other metals combined. Erythematous and eczematous eruptions, sometimes with lichenification, appear beneath earrings, bracelets, hairpins, rings, wristwatches, clasps, metallic eyeglass frames, and metal buttons. Exposure to the metal may not always be readily apparent. Sweat, blood, or saline may leach nickel from stainless steel. Diagnosis is established by a positive patch test to 2.5% nickel sulfate solution. Mercaptobenzothiazole (choice D) is a chemical used in the process of manufacturing rubber. Rubber dermatitis generally occurs on the hands from wearing gloves (in surgeons, nurses, homemakers). The eruption is usually sharply limited to the gloved area but may spread up the forearms. Natural and synthetic rubbers are used separately or in combination to make the final rubber product. It is the chemicals added in the rubber manufacturing process (the accelerators and antioxidants) that are the most common cause of allergic contact dermatitis. Elastic in the rubber in underwear is chemically transformed by laundry bleach into a potent sensitizing substance that induces allergic contact dermatitis at sites where it is close to the body: around the waist or under the bra straps. Clinical Pearls : Allergic contact dermatitis (ACD) refers to an inflammatory skin disease characterized by pruritic, eczematous lesion that are a delayed type hypersensitivity reaction mediated by T cells that occur after the skin is contacted by an offending chemical in patients who have been previously sensitized to the chemical. Allergic contact dermatitis (ACD) refers to an inflammatory skin disease characterized by pruritic, eczematous lesion that are a delayed type hypersensitivity reaction mediated by T cells that occur after the skin is contacted by an offending chemical in patients who have been previously sensitized to the chemical.

A 6-year-old girl with acute lymphocytic leukemia (ALL) is admitted to the hospital with a generalized vesicular rash and high fever. She has no previous history of chickenpox and has never received immunization for varicella. Her 4-year-old sister recently recovered from chickenpox that started about 10 days ago. On admission her temperature is 38.4º C (101.1º F), blood pressure 94/58 mm Hg, pulse 80/min, and respirations 20/min. Which of the following is the most serious complication of varicella that could occur in this setting? A. Arthritis B. Cellulitis C. Endocarditis D. Hepatitis E. Pneumonia

The correct answer is E. Varicella (chickenpox) usually presents with cutaneous infection in immunocompetent hosts. However, in immunocompromised patients, such as this girl with acute lymphocytic leukemia (ALL), varicella can cause serious and life-threatening complications, such as varicella pneumonia. Varicella pneumonia has a very poor prognosis in patients with ALL, reaching a mortality of roughly 25%. The clinical course usually deteriorates rapidly and many patients die within 3 days of the diagnosis. Pneumonia is exceedingly rare in normal children, but it can happen in older adults. IV acyclovir therapy is indicated in immunocompromised patients with varicella infection. Therapy should be initiated early in the disease course to maximize efficacy of the treatment. Varicella-zoster immune globulin (VZIG) may modify the disease course if it is given shortly after the exposure. It is not useful, however, once the disease is established, as in this clinical vignette. Children with varicella also should not be given salicylates because of a theoretically increased risk of subsequent Reye syndrome. Patients should be both contact- and airborne-isolated during the entire course of the illness. Less common complications include encephalitis, hepatitis, glomerulonephritis, arthritis, and thrombocytopenia. Clinical Pearls Varicella-zoster virus (VZV) is a herpes virus that has 2 clinical presentations: varicella (chickenpox) and herpes zoster (shingles). Chickenpox, or primary VZV infection, causes a diffuse vesicular rash. Children with chickenpox tend to have a milder disease as compared to adults or immunocompromised patients with chickenpox. Pneumonia is an uncommon complication in immunocompetent children, but it is a major cause of morbidity and mortality in adults with chickenpox. It should be noted that incidence of post-varicella pneumonia has decreased since the introduction of the varicella vaccine. Varicella pneumonia is more common in smokers, pregnant patients, immunocompromised patients, and male patients. Patients will typically present with symptoms of progressive tachypnea, dyspnea, dry cough, and possibly hemoptysis about 1-6 days after the rash appeared. Patients will have impaired gas exchange with progressive hypoxemia. Chest x-ray will likely show diffuse bilateral infiltrates or a nodular component early on in the illness which will later become calcified. Treatment with IV acyclovir should be initiated immediately when the diagnosis is suspected, as it has been associated with clinical improvement and resolution of the pneumonia. Secondary VZV infection or shingles represents an endogenous reactivation of latent VZV, which leads to a localized vesicular rash along a particular dermatome.


Conjuntos de estudio relacionados

How to calculate the standard deviation

View Set

International Management - Ch. 8

View Set

Chapter 44: Introduction to the Gastrointestinal System and Accessory Structures

View Set

NIC semester 3, ATI fundamentals

View Set

Biomed.- 4.2.3 Conclusion Questions

View Set

Unit 6: Lesson 1: LS Assignment 1

View Set